You are on page 1of 59

RMO–1990

1. Two boxes contain between them 65 balls of several different sizes. Each ball is white, black,
red or yellow. If you take any 5 balls of the same colour at least two of them will always be
of the same size (radius). Prove that there are at least 3 balls which lie in the same box have
the same colour and have the same size (radius).
2. For all positive real numbers a, b, c prove that
a b c 3
+ + ≥ .
b+c c+a a+b 2

3. A square sheet of paper ABCD is so folded that B falls on the mid-point M of CD. Prove
that the crease will divide BC in the ratio 5 : 3.
4. Find the remainder when 21990 is divided by 1990.
5. P is any point inside a triangle ABC. The perimeter of the triangle AB + BC + CA = 2s.
Prove that
s < AP + BP + CP < 2s.

6. N is a 50 digit number (in the decimal scale). All digits except the 26th digit (from the left)
are 1. If N is divisible by 13, find the 26th digit.
7. A censusman on duty visited a house which the lady inmates declined to reveal their individual
ages, but said — “we do not mind giving you the sum of the ages of any two ladies you may
choose”. Thereupon the censusman said — “In that case please give me the sum of the ages of
every possible pair of you”. The gave the sums as follows : 30, 33, 41, 58, 66, 69. The censusman
took these figures and happily went away. How did he calculate the individual ages of the ladies
from these figures.
8. If the circumcenter and centroid of a triangle coincide, prove that the triangle must be equi-
lateral.

1
RMO–1991
1. Let P be an interior point of a triangle ABC and AP , BP , CP meet the sides BC, CA, AB
in D, E, F respectively. Show that
AP AF AE
= + .
PD FB EC

2. If a, b, c and d are any 4 positive real numbers, then prove that


a b c d
+ + + ≥ 4.
b c d a

3. A four-digit number has the following properties :


(a) it is a perfect square,
(b) its first two digits are equal to each other,
(c) its last two digits are equal to each other.

Find all such four-digit numbers.


4. There are two urns each containing an arbitrary number of balls. Both are non-empty to begin
with. We are allowed two types of operations:
(a) remove an equal number of balls simultaneously from the urns, and
(b) double the number of balls in any one of them.
Show that after performing these operations finitely many times, both the urns can be made
empty.
5. Take any point P1 on the side BC of a triangle ABC and draw the following chain of lines :
P1 P2 parallel to AC (P2 on AB), P2 P3 parallel to BC, P3 P4 parallel to AB, P4 P5 parallel to
CA, and P5 P6 parallel to BC. Here P2 , P5 lie on AB; P3 , P6 lie on CA; and P4 on BC. Show
that P6 P1 is parallel to AB.
6. Find all integer values of a such that the quadratic expression

(x + a)(x + 1991) + 1

can be factored as a product (x + b)(x + c) where b and c are integers.


7. Prove that n4 + 4n is composite for all integer values of n greater than 1.
8. The 64 squares of an 8 × 8 chessboard are filled with positive integers in such a way that each
integer is the average of the integers on the neighbouring squares. (Two squares are neighbours
if they share a common edge or a common vertex. Thus a square can have 8, 5 or 3 neighbours
depending on its position). Show that all the 64 integer entries are in fact equal.

1
RMO–1992
1. Determine the set of integers n for which n2 + 19n + 92 is a square of an integer.
1 1 1
2. If + = , where a, b, c are positive integers with no common factor, prove that (a + b) is
a b c
the square of an integer.
2000
3. Determine the largest 3-digit prime factor of the integer C1000 .
4. ABCD is a cyclic quadrilateral with AC⊥BD; AC meets BD at E. Prove that

EA2 + EB 2 + EC 2 + ED2 = 4R2 ,

where R is the radius of the circumscribing circle.


5. ABCD is a cyclic quadrilateral; x, y, z are the distances of A from the lines BD, BC, CD
respectively. Prove that
BD BC CD
= + .
x y z
6. ABCD is a quadrilateral and P , Q are mid-points of CD, AB respectively. Let AP , DQ meet
at X, and BP , CQ meet at Y . Prove that

area of ADX + area of BCY = area of quadrilateral P XQY .

7. Prove that
1 1 1 1 1
1< + + + ... + <1 .
1001 1002 1003 3001 3
8. Solve the system

(x + y)(x + y + z) = 18
(y + z)(x + y + z) = 30
(z + x)(x + y + z) = 2A

in terms of the parameter A.


9. The cyclic octagon ABCDEF GH has sides a, a, a, a, b, b, b, b respectively. Find the radius
of the circle that circumscribes ABCDEF GH in terms of a and b.

1
RMO–1993
1. Let ABC be an acute-angled triangle and CD be the altitude through C. If AB = 8 and
CD = 6, find the distance between the mid-points of AD and BC.
2. Prove that the ten’s digit of any power of 3 is even. [e.g. the ten’s digit of 36 = 729 is 2].
3. Suppose A1 A2 . . . A20 is a 20-sided regular polygon. How many non-isosceles (scalene) triangles
can be formed whose vertices are among the vertices of the polygon but whose sides are not
the sides of the polygon?
4. Let ABCD be a rectangle with AB = a and BC = b. Suppose r1 is the radius of the circle
passing through A and B and touching CD; and similarly r2 is the radius of the circle passing
through B and C and touching AD. Show that
5
r1 + r2 ≥ (a + b).
8

5. Show that 1993 − 1399 is a positive integer divisible by 162.


6. If a, b, c, d are four positive real numbers such that abcd = 1, prove that

(1 + a)(1 + b)(1 + c)(1 + d) ≥ 16.

7. In a group of ten persons, each person is asked to write the sum of the ages of all the other
9 persons. If all the ten sums form the 9-element set {82, 83, 84, 85, 87, 89, 90, 91, 92} find the
individual ages of the persons (assuming them to be whole numbers of years).
8. I have 6 friends and during a vacation I met them during several dinners. I found that I dined
with all the 6 exactly on 1 day; with every 5 of them on 2 days; with every 4 of them on 3
days; with every 3 of them on 4 days; with every 2 of them on 5 days. Further every friend
was present at 7 dinners and every friend was absent at 7 dinners. How many dinners did I
have alone?

1
RMO–1994
1. A leaf is torn from a paperback novel. The sum of the numbers on the remaining pages is
15000. What are the page numbers on the torn leaf.
2. In the triangle ABC, the incircle touches the sides BC, CA and AB respectively at D, E and
F . If the radius of the incircle is 4 units and if BD, CE and AF are consecutive integers, find
the sides of the triangle ABC.
3. Find all 6-digit natural numbers a1 a2 a3 a4 a5 a6 formed by using the digits 1, 2, 3, 4, 5, 6, once
each such that the number a1 a2 . . . ak is divisible by k, for 1 ≤ k ≤ 6.
4. Solve the system of equations for real x and y :
 
1
5x 1 + 2 = 12
x + y2
 
1
5y 1 − 2 = 4.
x + y2

5. Let A be a set of 16 positive integers with the property that the product of any two distinct
numbers of A will not exceed 1994. Show that there are two numbers a and b in A which are
not relatively prime.
6. Let AC and BD be two chords of a circle with center O such that they intersect at right angles
inside the circle at the point M . Suppose K and L are the mid-points of the chord AB and
CD respectively. Prove that OKM L is a parallelogram.
7. Find the number of all rational numbers m/n such that
(a) 0 < m/n < 1
(b) m and n are relatively prime
(c) mn = 25!
8. If a, b and c are positive real numbers such that a + b + c = 1, prove that

(1 + a)(1 + b)(1 + c) ≥ 8(1 − a)(1 − b)(1 − c).

1
RMO–1995
1. In triangle ABC, K and L are points on the side BC (K being closer to B than L) such that
BC · KL = BK · CL and AL bisects 6 KAC. Show that AL is perpendicular to AB.
2. Call a positive integer n good if there are n integers, positive or negative, and not necessarily
distinct, such that their sum and product are both equal to n (e.g. 8 is good since

8 = 4 · 2 · 1 · 1 · 1 · 1(−1)(−1) = 4 + 2 + 1 + 1 + 1 + 1 + (−1) + (−1)).

Show that integers of the form 4k + 1 (k ≥ 0) and 4l (l ≥ 2) are good.


3. Prove that among any 18 consecutive 3-digit numbers there is at least one number which is
divisible by the sum of its digits.
4. Show that the quadratic expression

x2 + 7x − 14(q 2 + 1) = 0,

where q is an integer, has no integer root.


5. Show that for any triangle ABC, the following inequality is true :

a2 + b2 + c2 > 3 max{|a2 − b2 |, |b2 − c2 |, |c2 − a2 |},

where a, b, c are, as usual, the sides of the triangle.


6. Let A1 A2 A3 . . . A21 be a 21-sided regular polygon inscribed in a circle with center O. How
many triangles Ai Aj Ak , 1 ≤ i < j < k ≤ 21, contain the point O in their interior.
7. Show that for any real number x,
1
x2 sin x + x cos x + x2 + > 0.
2

1
RMO–1996
1. The sides of a triangle are three consecutive integers and its inradius is four units. Determine
the circumradius.
2. Find all triples (a, b, c) of positive integers such that
1 1 1
(1 + )(1 + )(1 + ) = 3.
a b c

3. Solve for real number x and y:

xy 2 = 15x2 + 17xy + 15y 2


x2 y = 20x2 + 3y 2 .

4. Suppose N is an n-digit positive integer such that

(a) all the n-digits are distinct; and


(b) the sum of any three consecutive digits is divisible by 5.

Prove that n is at most 6. Further, show that starting with any digit one can find a six-digit
number with these properties.
5. Let ABC be a triangle and ha the altitude through A. Prove that

(b + c)2 ≥ a2 + 4h2a .

(As usual a, b, c denote the sides BC, CA, AB respectively.)


6. Given any positive integer n show that there are two positive rational numbers a and b, a 6= b,
which are not integers and which are such that a − b, a2 − b2 , a3 − b3 , . . . , an − bn are all
integers.
7. If A is a fifty-element subset of the set {1, 2, 3, . . . , 100} such that no two numbers from A add
up to 100 show that A contains a square.

1
RMO–1997
1. Let P be an interior point of a triangle ABC and let BP and CP meet AC and AB in E and
F respectively. If [BP F ] = 4, [BP C] = 8 and [CP E] = 13, find [AF P E]. (Here [·] denotes
the area of a triangle or a quadrilateral, as the case may be.)
2. For each positive integer n, define an = 20 + n2 , and dn = gcd(an , an+1 ). Find the set of all
values that are taken by dn and show by examples that each of these values are attained.
3. Solve for real x:
1 1 1
+ = 9x) + ,
[x] [2x] 3
where [x] is the greatest integer less than or equal to x and (x) = x − [x], [e.g. [3.4] = 3 and
(3.4) = 0.4].
4. In a quadrilateral ABCD, it is given that AB is parallel to CD and the diagonals AC and
BD are perpendicular to each other.
Show that

(a) AD.BC ≥ AB.CD;


(b) AD + BC ≥ AB + CD.

5. Let x, y and z be three distinct real positive numbers. Determine with proof whether or not
the three real numbers
x y y z z x
| − |, | − |, | − |
y x z y x z
can be the lengths of the sides of a triangle.
6. Find the number of unordered pairs {A, B} (i.e., the pairs {A, B}and{B, A} are considered to
be the same) of subsets of an n-element set X which satisfy the conditions:

(a) A 6= B;
(b) A ∪ B = X

[e.g., if X = {a, b, c, d}, then {{a, b}, {b, c, d}}, {{a}, {b, c, d}}, {φ, {a, b, c, d}} are some of the
admissible pairs.]

1
RMO–1998
1. Let ABCD be a convex quadrilateral in which 6 BAC = 50◦ , 6 CAD = 60◦ , 6 CBD = 30◦ ,
and 6 BDC = 25◦ . If E is the point of intersection of AC and BD, find 6 AEB.
2. Let n be a positive integer and p1 , p2 , ·pn be n prime numbers all larger than 5 such that 6
divides p21 + p22 + ·p2n . Prove that 6 divides n.
3. Prove the following inequality for every natural number n:
1 1 1 1 1 1 1 1 1
(1 + + + · ) > ( + + + · ).
n+1 3 5 2n − 1 n 2 4 6 2n

4. Let ABC be a triangle with AB = BC and 6 BAC = 30◦ . Let A0 be the reflection of A in the
line BC; B 0 be the reflection of B in the line CA; C 0 be the reflection of C in the line AB.
Show that A0 , B 0 , C 0 form the vertices of an equilateral triangle.
5. Find the minimum possible least common multiple (lcm) of twenty (not necessarily distinct)
natural numbers whose sum is 801.
6. Given the 7-element set A = {a, b, c, d, e, f, g}, find a collection T of 3-element subsets of A
such that each pair of elements from A occurs exactly in one of the subsets of T .

1
RMO–1999
1. Prove that the inradius of a right-angled triangle with integer sides is an integer.
2. Find the number of positive integers which divide 10999 but not 10998 .
3. Let ABCD be a square and M, N points on sides AB, BC, respectably, such that 6 M DN =
45◦ . If R is the midpoint of M N show that RP = RQ where P, Q are the points of intersection
of AC with the lines M D, N D.
4. If p, q, r are the roots of the cubic equation x3 − 3px2 + 3q 2 x − r3 = 0, show that p = q = r.
5. If a, b, c are the sides of a triangle prove the following inequality:
a b c
+ + ≥ 3.
c+a−b a+b−c b+c−a

6. Find all solutions in integers m, n of the equation


4mn
(m − n)2 =
m + n − 1.

7. Find the number of quadratic polynomials, ax2 + bx + c, which satisfy the following conditions:
(a) a, b, c are distinct;
(b) a, b, c ∈ {1, 2, 3, . . . 1999} and
(c) x + 1 divides ax2 + bx + c.

1
Regional Mathematical Olympiad-2000
Problems and Solutions

1. Let AC be a line segment in the plane and B a point between A and C. Construct isosceles
triangles P AB and QBC on one side of the segment AC such that ∠AP B = ∠BQC = 120◦
and an isosceles triangle RAC on the otherside of AC such that ∠ARC = 120◦ . Show
that P QR is an equilateral triangle.

Solution: We give here 2 different solutions.

1. Drop perpendiculars from P and Q to AC and extend them to meet AR, RC in K, L


respectively. Join KB, P B, QB, LB, KL.(Fig.1.)
P K

Q P
Q
B
A C
A B C
L

K
R R
Fig. 1 Fig. 2
Observe that K, B, Q are collinear and so are P, B, L. ( This is because ∠QBC =
∠P BA = ∠KBA and similarly ∠P BA = ∠CBL.) By symmetry we see that
∠KP Q = ∠P KL and ∠KP B = ∠P KB. It follows that ∠LP Q = ∠LKQ and hence
K, L, Q, P are concyclic. We also note that ∠KP L + ∠KRL = 60◦ + 120◦ = 180◦ .
This implies that P, K, R, L are concyclic. We conclude that P, K, R, L, Q are con-
cyclic. This gives

∠P RQ = ∠P KQ = 60◦ , ∠RP Q = ∠RKQ = ∠RAP = 60◦ .

2. Produce AP and CQ to meet at K. Observe that AKCR is a rhombus and BQKP


is a parallelogram.(See Fig.2.) Put AP = x, CQ = y. Then P K = BQ = y,
KQ = P B = x and AR = RC = CK = KA = x + y. Using cosine rule in triangle
P KQ, we get P Q2 = x2 + y 2 − 2xy cos 120◦ = x2 + y 2 + xy. Similarly cosine rule in
triangle QCR gives QR2 = y 2 + (x + y)2 − 2xy cos 60◦ = x2 + y 2 + xy and cosine rule
in triangle P AR gives RP 2 = x2 + (x + y)2 − 2xy cos 60◦ = x2 + y 2 + xy. It follows
that P Q = QR = RP .

2. Solve the equation y 3 = x3 + 8x2 − 6x + 8, for positive integers x and y.

1
Solution: We have

y 3 − (x + 1)3 = x3 + 8x2 − 6x + 8 − (x3 + 3x2 + 3x + 1) = 5x2 − 9x + 7.

Consider the quadratic equation 5x2 − 9x + 7 = 0. The discriminant of this equation is


D = 92 − 4 × 5 × 7 = −59 < 0 and hence the expression 5x2 − 9x + 7 is positive for all real
values of x. We conclude that (x + 1)3 < y 3 and hence x + 1 < y.
On the other hand we have

(x + 3)3 − y 3 = x3 + 9x2 + 27x + 27 − (x3 + 8x2 − 6x + 8) = x2 + 33x + 19 > 0

for all positive x. We conclude that y < x + 3. Thus we must have y = x + 2. Putting
this value of y, we get

0 = y 3 − (x + 2)3 = x3 + 8x2 − 6x + 8 − (x3 + 6x2 + 12x + 8) = 2x2 − 18x.

We conclude that x = 0 and y = 2 or x = 9 and y = 11.

3. Suppose hx1 , x2 , . . . , xn , . . .i is a sequence of positive real numbers such that x1 ≥ x2 ≥


x3 ≥ · · · ≥ xn · · · , and for all n
x1 x4 x9 x 2
+ + + · · · + n ≤ 1.
1 2 3 n
Show that for all k the following inequality is satisfied:
x1 x2 x3 xk
+ + + ··· + ≤ 3.
1 2 3 k

Solution: Let k be a natural number and n be the unique integer such that (n − 1)2 ≤
k < n2 . Then we see that
k
X xr x
1 x2 x3   x4 x5 x8 
≤ + + + + + ··· +
r=1
r 1 2 3 4 5 8
x(n−1)2
 
xk xn2 −1
+··· + + ··· + + ··· + 2
(n − 1)2 k n −1
x x1 x1   x4 x4 x4 
1
≤ + + + + + ··· +
1 1 1 4 4  4
x(n−1)2 x(n−1)2

+··· + + ··· +
(n − 1)2 (n − 1)2
3x1 5x2 (2n − 1)x(n−1)2
= + + ··· +
1 4 (n − 1)2
n−1
X (2r + 1)xr2
=
r2
r=1

2
n−1
X 3r
≤ x2
r=1
r2 r
n−1
X xr2
= 3 ≤ 3,
r
r=1
where the last inequality follows from the given hypothesis.
4. All the 7-digit numbers containing each of the digits 1, 2, 3, 4, 5, 6, 7 exactly once, and not
divisible by 5, are arranged in the increasing order. Find the 2000-th number in this list.
Solution: The number of 7-digit numbers with 1 in the left most place and containing
each of the digits 1, 2, 3, 4, 5, 6, 7 exactly once is 6! = 720. But 120 of these end in 5 and
hence are divisible by 5. Thus the number of 7-digit numbers with 1 in the left most place
and containing each of the digits 1, 2, 3, 4, 5, 6, 7 exactly once but not divisible by 5 is 600.
Similarly the number of 7-digit numbers with 2 and 3 in the left most place and containing
each of the digits 1, 2, 3, 4, 5, 6, 7 exactly once but not divisible by 5 is also 600 each. These
account for 1800 numbers. Hence 2000-th number must have 4 in the left most place.
Again the number of such 7-digit numbers beginning with 41,42 and not divisible by 5 is
120 − 24 = 96 each and these account for 192 numbers. This shows that 2000-th number
in the list must begin with 43.
The next 8 numbers in the list are: 4312567, 4312576, 4312657, 4312756, 4315267, 4315276,
4315627 and 4315672. Thus 2000-th number in the list is 4315672.
5. The internal bisector of angle A in a triangle ABC with AC > AB, meets the circumcircle
Γ of the triangle in D. Join D to the centre O of the circle Γ and suppose DO meets AC
in E, possibly when extended. Given that BE is perpendicular to AD, show that AO is
parallel to BD.
Solution: We consider here the case when ABC is an acute-angled triangle; the cases
when ∠A is obtuse or one of the angles ∠B and ∠C is obtuse may be handled similarly.

N O
B C
M

3
Let M be the point of intersection of DE and BC; let AD intersect BE in N . Since M E
is the perpendicular bisector of BC, we have BE = CE. Since AN is the internal bisector
of ∠A, and is perpendicular to BE, it must bisect BE; i.e., BN = N E. This in turn
implies that DN bisects ∠BDE. But ∠BDA = ∠BCA = ∠C. Thus ∠ODA = ∠C. Since
OD = OA, we get ∠OAD = ∠C. It follows that ∠BDA = ∠C = ∠OAD. This implies
that OA is parallel to BD.

6. (i) Consider two positive integers a and b which are such that aabb is divisible by 2000.
What is the least possible value of the product ab?
(ii) Consider two positive integers a and b which are such that abba is divisible by 2000.
What is the least possible value of the product ab?

Solution: We have 2000 = 24 53 .


(i) Since 2000 divides aa bb , it follows that 2 divides a or b and similarly 5 divides a or b.
In any case 10 divides ab. Thus the least possible value of ab for which 2000|aa bb must
be a multiple of 10. Since 2000 divides 1010 11 , we can take a = 10, b = 1 to get the least
value of ab equal to 10.
(ii) As in (i) we conclude that 10 divides ab. Thus the least value of ab for which
2000|ab ba is again a multiple of 10. If ab = 10, then the possibilities are (a, b) =
(1, 10), (2, 5), (5, 2), (10, 1). But in all these cases it is easy to verify that 2000 does not
divide ab ba . The next multiple of 10 is 20. In this case we can take (a, b) = (4, 5) and
verify that 2000 divides 45 54 . Thus the least value here is 20.

7. Find all real values of a for which the equation x4 − 2ax2 + x + a2 − a = 0 has all its roots
real.

Solution: Let us consider x4 − 2ax2 + x + a2 − a = 0 as a quadratic equation in a. We


see that thee roots are
a = x2 + x, a = x2 − x + 1.
Thus we get a factorisation

(a − x2 − x)(a − x2 + x − 1) = 0.

It follows that x2 + x = a or x2 − x + 1 = a. Solving these we get


√ √
−1 ± 1 + 4a −1 ± 4a − 3
x= , or x = .
2 2
Thus all the four roots are real if and only if a ≥ 3/4.

4
Solutions for problems of CRMO-2001

1. Let BE and CF be the altitudes of an acute triangle ABC, with E on AC and F on AB.
Let O be the point of intersection of BE and CF . Take any line KL through O with K on
AB and L on AC. Suppose M and N are located on BE and CF respectively, such that
KM is perpendicular to BE and LN is perpendicular to CF . Prove that F M is parallel to
EN .

Solution: Observe that KM OF and ON LE are cyclic quadrilaterals. Hence

∠F M O = ∠F KO, and ∠OEN = ∠OLN.

E
F
O L
K

M N

B C

However we see that


π π
∠OLN = − ∠N OL = − ∠KOF = ∠OKF.
2 2
It follows that ∠F M O = ∠OEN . This forces that F M is parallel to EN .

2. Find all primes p and q such that p2 + 7pq + q 2 is the square of an integer.

Solution: Let p, q be primes such that p2 + 7pq + q 2 = m2 for some positive integer m. We
write
5pq = m2 − (p + q)2 = (m + p + q)(m − p − q).
We can immediately rule out the possibilities m + p + q = p, m + p + q = q and m + p + q = 5
(In the last case m > p, m > q and p, q are at least 2).
Consider the case m+p+q = 5p and m−p−q = q. Eliminating m, we obtain 2(p+q) = 5p−q.
It follows that p = q. Similarly, m + p + q = 5q and m − p − q = p leads to p = q. Finally
taking m + p + q = pq, m − p − q = 5 and eliminating m, we obtain 2(p + q) = pq − 5. This
can be reduced to (p − 2)(q − 2) = 9. Thus p = q = 5 or (p, q) = (3, 11), (11, 3). Thus the
set of solutions is
{(p, p) : p is a prime } ∪ {(3, 11), (11, 3)}.
3. Find the number of positive integers x which satisfy the condition
hxi h x i
= .
99 101
(Here [z] denotes, for any real z, the largest integer not exceeding z; e.g. [7/4] = 1.)
hxi h x i
Solution: We observe that = = 0 if and only if x ∈ {1, 2, 3, . . . , 98}, and
99 101
hxi h x i
there are 98 such numbers. If we want = = 1, then x should lie in the set
99 101 hxi h x i
{101, 102, . . . , 197}, which accounts for 97 numbers. In general, if we require = =
99 101
k, where k ≥ 1, then x must be in the set {101k, 101k + 1, . . . , 99(k + 1) − 1}, and there
are 99 − 2k such numbers. Observe that this set is not empty only if 99(k + 1) − 1 ≥ 101k
and this
h xrequirement
i h x i is met only if k ≤ 49. Thus the total number of positive integers x for
which = is given by
99 101
49
X
98 + (99 − 2k) = 2499.
k=1
– » – »
x x
[Remark: For any m ≥ 2 the number of positive integers x such that =
m−1 m+1
2 2
m −4 m −5
is if m is even and if m is odd.]
4 4
4. Consider an n × n array of numbers:
0 1
a11 a12 a13 ··· a1n
Ba
B 21 a22 a23 ··· a2n C
C
B . .. C
B . C
@ . . A
an1 an2 an3 ··· ann

Suppose each row consists of the n numbers 1, 2, 3, . . . , n in some order and aij = aji for
i = 1, 2, . . . , n and j = 1, 2, . . . , n. If n is odd, prove that the numbers a11 , a22 , a33 , . . . , ann
are 1, 2, 3, . . . , n in some order.

Solution: Let us see how many times a specific term, say 1, occurs in the matrix. Since
1 occurs once in each row, it occurs n times in the matrix. Now consider its occurrence off
the main diagonal. For each occurrence of 1 below the diagonal, there is a corresponding
occurrence above it, by the symmetry of the array. This accounts for an even number of
occurrences of 1 off the diagonal. But 1 occurs exactly n times and n is odd. Thus 1 must
occur at least once on the main diagonal. This is true of each of the numbers 1, 2, 3, . . . , n.
But there are only n numbers on the diagonal. Thus each of 1, 2, 3, . . . , n occurs exactly once
on the main diagonal. This implies that a11 , a22 , a33 , . . . , ann is a permutation of 1, 2, 3, . . . , n.

5. In a triangle ABC, D is a point on BC such that AD is the internal bisector of ∠A. Suppose
∠B = 2∠C and CD = AB. Prove that ∠A = 72◦ .

Solution 1.: Draw the angle bisector BE of ∠ABC to meet AC in E. Join ED. Since
∠B = 2∠C, it follows that ∠EBC = ∠ECB. We obtain EB = EC.
A

β β

E

α β
α α

B D C
Consider the triangles BEA and CED. We observe that BA = CD, BE = CE and
∠EBA = ∠ECD. Hence BEA ≡ CED giving EA = ED. If ∠DAC = β, then we obtain
∠ADE = β. Let I be the point of intersection of AD and BE. Now consider the triangles
AIB and DIE. They are similar since ∠BAI = β = ∠IDE and ∠AIB = ∠DIE. It follows
that ∠DEI = ∠ABI = ∠DBI. Thus BDE is isoceles and DB = DE = EA. We also
observe that ∠CED = ∠EAD + ∠EDA = 2β = ∠A. This implies that ED is parallel to
AB. Since BD = AE, we conclude that BC = AC. In particular ∠A = 2∠C. Thus the
total angle of ABC is 5∠C giving ∠C = 36◦ . We obtain ∠A = 72◦ .
Solution 2. We make use of the charectarisation: in a triangle ABC, ∠B = 2∠C if and
only if b2 = c(c + a). Note that CD = c and BD = a − c. Since AD is the angle bisector,
we also have
a−c c
= .
c b
This gives c2 = ab − bc and hence b2 = ca + ab − bc. It follows that b(b + c) = a(b + c) so
that a = b. Hence ∠A = 2∠C as well and we get ∠C = 36◦ . In turn ∠A = 72◦ .

6. If x, y, z are the sides of a triangle, then prove that

|x2 (y − z) + y 2 (z − x) + z 2 (x − y)| < xyz.

Solution: The given inequality may be written in the form

|(x − y)(y − z)(z − x)| < xyz.

Since x, y, z are the sides of a triangle, we know that |x − y| < z, |y − z| < x and |z − x| < y.
Multiplying these, we obtain the required inequality.

7. Prove that the product of the first 200 positive even integers differs from the product of the
first 200 positive odd integers by a multiple of 401.
Solution: We have to prove that

401 divides 2 · 4 · 6 · · · · · 400 − 1 · 3 · 5 · · · · · 399.

Write x = 401. Then this difference is equal to

(x − 1)(x − 3) · · · (x − 399) − 1 · 3 · 5 · · · · · 399.

If we expand this as a polynomial in x, the constant terms get canceled as there are even
number of odd factors ((−1)200 = 1). The remaining terms are integral multiples of x and
hence the difference is a multiple of x. Thus 401 divides the above difference.
Problems and Solutions. . . CRMO-2002
1. In an acute triangle ABC, points D, E, F are located on the sides BC, CA, AB respectively
such that
CD CA AE AB BF BC
= , = , = .
CE CB AF AC BD BA
Prove that AD, BE, CF are the altitudes of ABC.

Solution: Put CD = x. Then with usual notations we get


CD · CB ax
CE = = .
CA b
Since AE = AC − CE = b − CE, we obtain

b2 − ax AE · AC b2 − ax
AE = , AF = = .
b AB c

A A

α E E
β

α β
α
F F

B D C B D C

Fig. 1 Fig. 2

This in turn gives


c2 − b2 + ax
BF = AB − AF = .
c
Finally we obtain
c2 − b2 + ax
BD = .
a
Using BD = a − x, we get
a 2 − c2 + b 2
x= .
2a
However, if L is the foot of perpendicular from A on to BC then, using Pythagoras theorem
in triangles ALB and ALC we get

b2 − LC 2 = c2 − (a − LC)2

which reduces to LC = (a2 − c2 + b2 )/2a. We conclude that LC = DC proving L = D. Or, we


can also infer that x = b cos C from cosine rule in triangle ABC. This implies that CD = CL,
since CL = b cos C from right triangle ALC. Thus AD is altitude on to BC. Similar proof
works for the remaining altitudes.
Alternately, we see that CD ·CB = CE ·CA, so that ABDE is a cyclic quadrilateral. Similarly
we infer that BCEF and CAF D are also cyclic quadrilaterals. (See Fig. 2.) Thus ∠AEF =
∠B = ∠CED. Moreover ∠BED = ∠DAF = ∠DCF = ∠BCF = ∠BEF . It follows that
∠BEA = ∠BEC and hence each is a right angle thus proving that BE is an altitude. Similarly
we prove that CF and AD are altitudes. (Note that the concurrence of the lines AD, BE,
CF are not required.)
2. Solve the following equation for real x:

(x2 + x − 2)3 + (2x2 − x − 1)3 = 27(x2 − 1)3 .

Solution: By setting u = x2 + x − 2 and v = 2x2 − x − 1, we observe that the equation reduces


to u3 + v 3 = (u + v)3 . Since (u + v)3 = u3 + v 3 + 3uv(u + v), it follows that uv(u + v) = 0.
Hence u = 0 or v = 0 or u + v = 0. Thus we obtain x2 + x − 2 = 0 or 2x2 − x − 1 = 0 or
x2 − 1 = 0. Solving each of them we get x = 1, −2 or x = 1, −1/2 or x = 1, −1. Thus x = 1 is
a root of multiplicity 3 and the other roots are −1, −2, −1/2.

(Alternately, it can be seen that x − 1 is a factor of x2 + x − 2, 2x2 − x − 1 and x2 − 1. Thus


we can write the equation in the form

(x − 1)3 (x + 2)3 + (x − 1)3 (2x + 1)3 = 27(x − 1)3 (x + 1)3 .

Thus it is sufficient to solve the cubic equation

(x + 2)3 + (2x + 1)3 = 27(x + 1)3 .

This can be solved as earlier or expanding every thing and simplifying the relation.)
3. Let a, b, c be positive integers such that a divides b2 , b divides c2 and c divides a2 . Prove that
abc divides (a + b + c)7 .

Solution: Consider the expansion of (a + b + c)7 . We show that each term here is divisible
by abc. It contains terms of the form rklm ak bl cm , where rklm is a constant( some binomial
coefficient) and k, l, m are nonnegative integers such that k + l + m = 7. If k ≥ 1, l ≥ 1, m ≥ 1,
then abc divides ak bl cm . Hence we have to consider terms in which one or two of k, l, m are
zero. Suppose for example k = l = 0 and consider c7 . Since b divides c2 and a divides c4 ,
it follows that abc divides c7 . A similar argument gives the result for a7 or b7 . Consider the
case in which two indices are nonzero, say for example, bc6 . Since a divides c4 , here again abc
divides bc6 . If we take b2 c5 , then also using a divides c4 we obtain the result. For b3 c4 , we use
the fact that a divides b2 . Similar argument works for b4 c3 , b5 c2 and b6 c. Thus each of the
terms in the expansion of (a + b + c)7 is divisible by abc.
4. Suppose the integers 1, 2, 3, . . . , 10 are split into two disjoint collections a1 , a2 , a3 , a4 , a5 and
b1 , b2 , b3 , b4 , b5 such that
a1 < a2 < a3 < a4 < a5 ,
b1 > b2 > b3 > b4 > b5 .
(i) Show that the larger number in any pair {aj , bj }, 1 ≤ j ≤ 5, is at least 6.
(ii) Show that |a1 − b1 | + |a2 − b2 | + |a3 − b3 | + |a4 − b4 | + |a5 − b5 | = 25 for every such partition.

2
Solution:
(i) Fix any pair {aj , bj }. We have a1 < a2 < · · · < aj−1 < aj and bj > bj+1 > · · · > b5 . Thus
there are j − 1 numbers smaller than aj and 5 − j numbers smaller than bj . Together
they account for j − 1 + 5 − j = 4 distinct numbers smaller than aj as well as bj . Hence
the larger of aj and bj is at least 6.
(ii) The first part shows that the larger numbers in the pairs {aj , bj }, 1 ≤ j ≤ 5, are
6, 7, 8, 9, 10 and the smaller numbers are1, 2, 3, 4, 5. This implies that

|a1 − b1 | + |a2 − b2 | + |a3 − b3 | + |a4 − b4 | + |a5 − b5 |


= 10 + 9 + 8 + 7 + 6 − (1 + 2 + 3 + 4 + 5) = 25.

5. The circumference of a circle is divided into eight arcs by a convex quadrilateral ABCD, with
four arcs lying inside the quadrilateral and the remaining four lying outside it. The lengths of
the arcs lying inside the quadrilateral are denoted by p, q, r, s in counter-clockwise direction
starting from some arc. Suppose p + r = q + s. Prove that ABCD is a cyclic quadrilateral.

Solution: Let the lengths of the arcs XY , U V , EF , GH be respectively p, q, r, s. We also


the following notations: (See figure)

D G
δ_1
F
C
δ_4
γ_2 γ_1

δ_2 γ_4 E
H
δ_3 γ_3

P β_3
α_3

β_2 V
β_4
β_1
α_4
X U B
α_2
α_1

A Y
Fig. 3

∠XAY = α1 , ∠AY P = α2 , ∠Y P X = α3 , ∠P XA = α4 , ∠U BY = β1 , ∠BV P = β2 , ∠V P U =


β3 , ∠P U B = β4 , ∠ECF = γ1 , ∠CF P = γ2 , ∠F P E = γ3 , ∠P EC = γ4 , ∠GDH = δ1 ,
∠DHP = δ2 , ∠HP G = δ3 , ∠P GD = δ4 .
We observe that X X X X
αj = βj = γj = δj = 2π.
It follows that X X
(αj + γj ) = (βj + δj ).
On the other hand, we also have α2 = β4 since P Y = P U . Similarly we have other relations:
β2 = γ4 , γ2 = δ4 and δ2 = α4 . It follows that

α1 + α3 + γ1 + γ3 = β1 + β3 + δ1 + δ3 .

3
But p + r = q + s implies that α3 + γ3 = β3 + δ3 . We thus obtain

α1 + γ1 = β1 + δ1 .

Since α1 + γ1 + β1 + δ1 = 360◦ , it follows that ABCD is a cyclic quadrilateral.


6. For any natural number n > 1, prove the inequality:
1 1 2 3 n 1 1
< 2 + + + ··· + 2 < + .
2 n + 1 n2 + 2 n2 + 3 n +n 2 2n

Solution: We have n2 < n2 + 1 < n2 + 2 < n2 + 3 · · · < n2 + n. Hence we see that


1 2 n 1 2 n
+ + ··· + 2 > + + ···+ 2
n2 + 1 n2 + 2 n +n n2 + n n2 + n n +n
1 1
= 2
(1 + 2 + 3 + · · · n) = .
n +n 2
Similarly, we see that
1 2 n 1 2 n
+ + ···+ 2 < + 2 + ···+ 2
n2 + 1 n2 + 2 n +n n2 n n
1 1 1
= 2
(1 + 2 + 3 + · · · n) = + .
n 2 2n

7. Find all integers a, b, c, d satisfying the following relations:


(i) 1 ≤ a ≤ b ≤ c ≤ d;
(ii) ab + cd = a + b + c + d + 3.

Solution: We may write (ii) in the form

ab − a − b + 1 + cd − c − d + 1 = 5.

Thus we obtain the equation (a − 1)(b − 1) + (c − 1)(d − 1) = 5. If a − 1 ≥ 2, then (i) shows


that b − 1 ≥ 2, c − 1 ≥ 2 and d − 1 ≥ 2 so that (a − 1)(b − 1) + (c − 1)(d − 1) ≥ 8. It follows
that a − 1 = 0 or 1.
If a − 1 = 0, then the contribution from (a − 1)(b − 1) to the sum is zero for any choice of b.
But then (c − 1)(d − 1) = 5 implies that c − 1 = 1 and d − 1 = 5 by (i). Again (i) shows that
b − 1 = 0 or 1 since b ≤ c. Taking b − 1 = 0, c − 1 = 1 and d − 1 = 5 we get the solution
(a, b, c, d) = (1, 1, 2, 6). Similarly, b−1 = 1, c−1 = 1 and d−1 = 5 gives (a, b, c, d) = (1, 2, 2, 6).
In the other case a − 1 = 1, we see that b − 1 = 2 is not possible for then c − 1 ≥ 2 and
d − 1 ≥ 2. Thus b − 1 = 1 and this gives (c − 1)(d − 1) = 4. It follows that c − 1 = 1,
d − 1 = 4 or c − 1 = 2, d − 1 = 2. Considering each of these, we get two more solutions:
(a, b, c, d) = (2, 2, 2, 5), (2, 2, 3, 3).
It is easy to verify all these four quadruples are indeed solutions to our problem.

4
Solutions to CRMO-2003

1. Let ABC be a triangle in which AB = AC and ∠CAB = 90◦ . Suppose M and


N are points on the hypotenuse BC such that BM 2 + CN 2 = M N 2 . Prove that
∠M AN = 45◦ .

Solution:
Draw CP perpendicular to CB and BQ perpendicular to CB such that CP = BM ,
BQ = CN . Join P A, P M , P N , QA, QM , QN . (See Fig. 1.)

C

P N

α
α

M
α 2β

A B

α
Q
Fig. 1.

In triangles CP A and BM A, we have ∠P CA = 45◦ = ∠M BA; P C = M B, CA =


BA. So △CP A ≡ △BM A. Hence ∠P AC = ∠BAM = α, say. Consequently,
∠M AP = ∠BAC = 90◦ , whence P AM C is a cyclic quadrilateral. Therefore ∠P M C =
∠P AC = α. Again P N 2 = P C 2 + CN 2 = BM 2 + CN 2 = M N 2 . So P N = M N , giv-
ing ∠N P M = ∠N M P = α, in △P M N . Hence ∠P N C = 2α. Likewise ∠QM B = 2β,
where β = ∠CAN . Also △N CP ≡ △QBM , as CP = BM, N C = BQ and
∠N CP = 90◦ = ∠QBM . Therefore, ∠CP N = ∠BM Q = 2β, whence 2α + 2β = 90◦ ;
α + β = 45◦ ; finally ∠M AN = 90◦ − (α + β) = 45◦ .

Aliter: Let AB = AC = a, so that BC = 2a; and ∠M AB = α, ∠CAN = β.(See
Fig. 2.)
By the Sine Law, we have from △ABM that
BM AB
= .
sin α sin(α + 45◦ )
√ √
a 2 sin α a 2u
So BM = = , where u = tan α.
cos α + sin α 1+u

N
π/4

a
β
M

π/4
α

A a B

Fig. 2.

a 2v
Similarly CN = , where v = tan β. But
1+v

BM 2 + CN 2 = M N 2 = (BC − M B − N C)2
= BC 2 + BM 2 + CN 2
− 2BC · M B − 2BC · N C + M B · N C.

So
BC 2 − 2BC · M B − 2BC · N C + 2M B · N C = 0.
This reduces to
√ √
2
√ a 2u √ a 2v 4a2 uv
2a − 2 2a − 2 2a + = 0.
1+u 1+v (1 + u)(1 + v)

Multiplying by (1 + u)(1 + v)/2a2 , we obtain

(1 + u)(1 + v) − 2u(1 + v) − 2v(1 + u) + 2uv = 0.

Simplification gives 1 − u − v − uv = 0. So
u+v
tan(α + β) = = 1.
1 − uv
This gives α + β = 45◦ ,whence ∠M AN = 45◦ , as well.

2
  h i  
n n  n
2. If n is an integer greater than 7, prove that − is divisible by 7. Here
7 7 7
denotes the number of ways of choosing 7 objects from among nobjects; also, for any
real number x, [x] denotes the greatest integer not exceeding x.

Solution: We have  
n n(n − 1)(n − 2) . . . (n − 6)
= .
7 7!
In the numerator, there is a factor divisible by 7, and the other six factors leave the
remainders 1,2,3,4,5,6 in some order when divided by 7.
Hence the numerator may be written as

7k · (7k1 + 1) · (7k2 + 2) · · · (7k6 + 6).


hni 
Also we conclude that = k, as in the set n, n − 1, . . . n − 6 , 7k is the only
p
number which is a multiple of 7. If the given number is called Q, then
(7k1 + 1)(7k2 + 2) . . . (7k6 + 6)
Q = 7k · −k
 7! 
(7k1 + 1) . . . (7k6 + 6) − 6!
= k
6!
k[7t + 6! − 6!]
=
6!
7tk
= .
6!
We know that Q is an integer, and so 6! divides 7tk. Since gcd(7, 6!) = 1, even after
cancellation there is a factor of 7 still left in the numerator. Hence 7 divides Q, as
desired.

3. Let a, b, c be three positive real numbers such that a + b + c = 1. Prove that among
the three numbers a − ab, b − bc, c − ca there is one which is at most 1/4 and there
is one which is at least 2/9.
Solution: By AM-GM inequality, we have

a+1−a 2 1
 
a(1 − a) ≤ = .
2 4
Similarly we also have
1 1
b(1 − b) ≤ and c(1 − c) ≤ .
4 4
Multiplying these we obtain
1
abc(1 − a)(1 − b)(1 − c) ≤ .
43

3
We may rewrite this in the form
1
a(1 − b) · b(1 − c) · c(1 − a) ≤ .
43

Hence one factor at least among a(1 − b), b(1 − c), c(1 − a) has to be less than or
1 1
equal to ; otherwise lhs would exceed 3 .
4 4
Again consider the sum a(1−b)+b(1−c)+c(1−a). This is equal to a+b+c−ab−bc−ca.
We observe that  2
3 ab + bc + ca ≤ a + b + c ,
which, in fact, is equivalent to (a − b)2 + (b − c)2 + (c − a)2 ≥ 0. This leads to the
inequality
1 1 2
a + b + c − ab − bc − ca ≥ (a + b + c) − (a + b + c)2 = 1 − = .
3 3 3

Hence one summand at least among a(1 − b), b(1 − c), c(1 − a) has to be greater
2 2
than or equal to ; otherwise lhs would be less than .
9 3
4. Find the number of ordered triples (x, y, z) of nonnegative integers satisfying the
conditions:

(i) x ≤ y ≤ z;
(ii) x + y + z ≤ 100.

Solution: We count by brute force considering the cases x = 0, x = 1, . . . , x = 33.


Observe that the least value x can take is zero, and its largest value is 33.
 
x = 0 If y = 0, then z ∈ 0, 1, 2, . . . , 100 ; if y=1, then z ∈ 1, 2, . . . , 99 ; if y = 2,
then z ∈ 2, 3, . . . , 98 ; and so on. Finally if y = 50, then z ∈ {50}. Thus there are
altogether 101 + 99 + 97 + · · · + 1 = 512 possibilities.

x
 = 1. Observe that y ≥ 1. If y =
 1, then z ∈ 1, 2, . . . , 98 ; if y = 2, then z ∈
2, 3,
 . . . , 97 ; if y = 3, then z ∈ 3, 4, . . . , 96 ; and so on. Finally if y = 49, then
z ∈ 49, 50 . Thus there are altogether 98 + 96 + 94 + · · · + 2 = 49 · 50 possibilities.
General
 case. Let x be even, say, x = 2k, 0 ≤ k ≤ 16. If y = 2k, then z ∈
2k, 2k + 1, . . . , 100 − 4k ; if y = 2k + 1, then z ∈ 2k + 1, 2k + 2, . . . , 99 − 4k ; if
y = 2k + 2, then z ∈ 2k + 2, 2k + 3, . . . , 99 − 4k ; and so on.

Finally, if y = 50 − k, then z ∈ 50 − k . There are altogether

(101 − 6k) + (99 − 6k) + (97 − 6k) + · · · + 1 = (51 − 3k)2

possibilities.

4

Let x be odd, say, x = 2k + 1, 0 ≤ k ≤16. If y = 2k + 1, then z ∈ 2k + 1, 2k +
2, . . . , 98 −
 4k ; if y = 2k + 2, then z ∈ 2k + 2, 2k + 3, . . . , 97 − 4k ; if y = 2k + 3,
then z ∈ 2k + 3, 2k + 4, . . . , 96 − 4k ; and so on.

Finally, if y = 49 − k, then z ∈ 49 − k, 50 − k . There are altogether

(98 − 6k) + (96 − 6k) + (94 − 6k) + · · · + 2 = (49 − 3k)(50 − 3k)

possibilities.
The last two cases would be as follows:
 
x = 32: if y = 32, then z ∈ 32, 33, 34, 35, 36 ; if y = 33, then z ∈ 33, 34, 35 ; if
2

y = 34, then z ∈ 34 ; altogether 5 + 3 + 1 = 9 = 3 possibilities.

x = 33: if y = 33, then z ∈ 33, 34 ; only 2=1.2 possibilities.
Thus the total number of triples, say T, is given by,
16
X 16
X
2
T = (51 − 3k) + (49 − 3k)(50 − 3k).
k=0 k=0

Writing this in the reverse order, we obtain


17
X 17
X
T = (3k)2 + (3k − 2)(3k − 1)
k=1 k=0
17
X X17
= 18 k2 − 9 k + 34
k=1 k=1
   
17 · 18 · 35 17 · 18
= 18 −9 + 34
6 2
= 30, 787.

Thus the answer is 30787.

Aliter
It is known that the number of ways in which a given positive integer n ≥ 3 can be
expressed as a sum of three positive
 2  integers x, y, z (that is, x + y + z = n), subject
n
to the condition x ≤ y ≤ z is , where {a} represents the integer closest to a. If
12
(n + 3)2
 
zero values are allowed for x, y, z then the corresponding count is , where
12
now n ≥ 0.

Since in our problem n = x + y + z ∈ 0, 1, 2, . . . , 100 , the desired answer is
100 
(n + 3)2
X 
.
n=0
12

5
For n = 0, 1, 2, 3, . . . , 11, the corrections for {} to get the nearest integers are
3 −4 −1 −1 −4 3 −4 −1 −1 −4
, , , 0, , , , , , 0, , .
12 12 12 12 12 12 12 12 12 12
So, for 12 consecutive integer values of n, the sum of the corrections is equal to
 
3−4−1−0−1−4−3 −7
×2= .
12 6

101 5 
Since = 8 + , there are 8 sets of 12 consecutive integers in 3,4,5, ... ,103
12 12
with 99,100,101,102,103 still remaining. Hence the total correction is
 
−7 3−4−1−0−1 −28 1 −115
×8+ = − = .
6 12 3 4 12

So the desired number T of triples (x, y, z) is equal to


100
X (n + 3)2 115
T = −
12 12
n=0
12
+ 22 + 32 + · · · + 1032 − 12 + 22
 
115
= −
12 12
103 · 104 · 207 5 115
= − −
6 · 12 12 12
= 30787.

5. Suppose P is an interior point of a triangle ABC such that the ratios

d(A, BC) d(B, CA)d(C, AB)


, ,
d(P, BC) d(P, CA)d(P, AB)

are all equal. Find the common value of these ratios. Here d(X, Y Z) denotes the
perpendicular distance from a point X to the line Y Z.
Solution: Let AP, BP, CP when extended, meet the sides BC, CA, AB in D, E, F
respectively. Draw AK, P L perpendicular to BC with K, L on BC.(See Fig. 3.)

6
y

A A (h,k)

F E
P
P(u,v)

x
B (0,0) C (a,0)
B K L D C

Fig. 3. Fig. 4.

Now
d(A, BC) AK AD
= = .
d(P, BC) PL PD
Similarly,
d(B, CA) BE d(C, AB) CF
= and = .
d(P, CA) PE d(P, AB) PF
So, we obtain
AD BE CF AP BP CP
= = , and hence = = .
PD PE PF PD PE PF
AP BP
From = and ∠AP B = ∠DP E, it follows that triangles AP B and DP E are
PD PE
similar. So ∠ABP = ∠DEP and hence AB is parallel to DE.
Similarly, BC is parallel to EF and CA is parallel to DF . Using these we obtain
BD AE AF DC
= = = ,
DC EC FB BD
whence BD2 = CD2 or which is same as BD = CD. Thus D is the midpoint of BC.
Similarly E, F are the midpoints of CA and AB respectively.
We infer that AD, BE, CF are indeed the medians of the triangle ABC and hence P
is the centroid of the triangle. So
AD BE CF
= = = 3,
PD PE PF
and consequently each of the given ratios is also equal to 3.

Aliter

7
(I) either (a − 5)2 > 4 and (a + 1)2 = 4;
(II) or (a − 5)2 = 4 and (a + 1)2 > 4.
Case (I) From (a+1)2 = 4, we have a = 1 or -3. But only a = 1 satisfies (a−5)2 > 4.

Thus a = 1. Also when a = 1,√equation (1) has solutions x = 2 + 3; and (2) has
solutions x = −1, −1. As 2 ± 3 > 0 and −1 < 0, we see that a = 1 is indeed a
solution.
Case (II) From (a − 5)2 = 4, we have a=3 or 7. Both these values of a satisfy the
inequality (a + 1)2 > 4.√When a = 3, equation (1)
√ has solutions x = 1, 1 and (2) has
the solutions x = −2 ± 3. As 1 > 0 and −2 ± 3 < 0, we see that a = 3 is in fact a
solution.
When a = 7, equation (1) has solutions x = −1, −1, which are negative contradicting
x ≥ 0.
Thus a = 1, a = 3 are the two desired values.

7. Consider the set X = 1, 2, 3, . . . , 9, 10 . Find two disjoint nonempty subsets A and
B of X such that
(a) A ∪ B = X;
(b) prod(A) is divisible by prod(B), where for any finite set of numbers C, prod(C)
denotes the product of all numbers in C ;
(c) the quotient prod(A)/prod(B) is as small as possible.

Solution: The prime factors of the numbers in set 1,2,3, . . . , 9,10 are 2,3,5,7. Also
only 7 ∈ X has the prime factor 7. Hence it cannot appear in B. For otherwise, 7 in
the denominator would not get canceled. Thus 7 ∈ A.
Hence
prod(A)/prod(B) ≥ 7.
The numbers having prime factor 3 are 3,6,9. So 3 and 6 should belong to one of A
and B, and 9 belongs to the other. We may take 3, 6 ∈ A, 9 ∈ B.
Also 5 divides 5 and 10. We take 5 ∈ A, 10 ∈ B. Finally we take 1, 2, 4 ∈ A, 8 ∈ B.
Thus  
A = 1, 2, 3, 4, 5, 6, 7 , B = 8, 9, 10 ,
so that
prod(A) 1·2·3·4·5·6·7
= = 7.
prod(B) 8 · 9 · 10
prod(A)
Thus 7 is the minimum value of . There are other possibilities for A and
prod(B) 
B:
 e.g., 1 may belong to either A or B. We may take A = 3, 5, 6, 7, 8 , B =
1, 2, 4, 9, 10 .

9
 
!#"%$!&('*),+.-0/1)2&43 56-768:9;&6-<9>=?)@/ =?82-ACB)@3 5D=?-?&E3 -0/FG9H&6+49>82)2&6-<IJ&6$H3K)2&L3 -M/ '*-?=03 )2&6N>=?)@/ =?82-OAJ1PQ/ $;R!-3 569;3K3 5(-0/ -):'
9S6&():TLS(-7$!)2&E3VUW$!&X3 5(-17-0/ 7-?&Y+.)2=?S68:9;/V+L/9ZB&\[]/ $!^_F`3 $O3 56-K8:)2&(-<Iba6'*S(=M5X3 569;3[]$H/K9H&EcX7$H)2&L3Vd`$H&X3 56-182)2&6-
Ife d1U`/ -?7./ -?'*-?&E3 '3 56-82-?&(NH3 5g$;[h3 56-K39;&6NH-?&E3[,/ $!^id`3 $<3 5(-1=?)@/ =?82-A
jlkmonqpZr]ksut

Y
O

Q y

X z
u
x
u

v -03wyx-K3 56-K[]$E$H3$H[z3 56-7-0/ 7-0&Y+()2=?S(8:9;/[]/ $H^iF`3 $3 56-V8:)2&(-IfaY9;&Y+>{gx-K3 56-K8:-0&6NH3 5\$H[h3 5(-K39H&(N!-?&E3wK|}[]/ $!^
l R t P

w`3 $\=?)@/ =?82-A ~K&DF1wW39H!-9€7$!)2&E3U'*S6=5D3 569;3Uwƒ‚ƒ{zK„ -<'*5($ZB`3 5Y9†31U)2'K3 5(-+.-?'*)@/ -Z+47$!)2&E3Z1‡z$#3 5():'
-?&Y+aL39Hˆ-19;&Ec\7$H)2&L3Vd`$H&g82):&(-IQ9H&6+\82-03‰€x-3 56-82-?&(NH3 5g$;[h3 56-K39;&6NH-?&E3KdŠ‹[]/ $H^id`3 $#A
Œ S(/*3 5(-0/V8:-M3Ox-3 56-/9!+.):S('K$;[u3 5(-=0)2/ =08:-AŽ9H&Y+482-03K‰Xx-3 56-8:-0&6NH3 5D$;[ 3 5(-39;&6NH-?&E31dŠ[,/ $!^dW3 $>A K!$!)2&
F1d‘e Ud v -03<Udi‚G’ze“F1d_‚i”.e wdi‚•0 Œ / $H^/ ):NH5E3O9;&6N!82-Z+–3*/ ),9;&6NH8:-0'Od1FŠ%e F|gdeMF1wd‘e d1UwB%-€5Y9ZRˆ-
/ -?'*7-?=03 )2Rˆ-082c—”E˜€‚i!˜™‰(˜;e F1w1˜>‚Gˆ˜™{˜He ”E˜€‚iF1w1˜™š• ˜€‚i!˜™{˜™š• ˜;e ’˜€‚G{˜™š• ˜;œ›L$gB‘-€$!x.39H)2&
‰ ˜ ‚`” ˜‘  ˜ ‚ƒ ˜ ™{ ˜ ™Ž• ˜  ˜ ‚ƒ{ ˜ ™C• ˜ ‚y’ ˜ KžV-?&6=0-‰>‚Ÿ’zV‡‘56)2'KNH):R!-?'dŠ‚`d| B56)2=5–)2'B5Y9;3VB‘-
&6-?-?+(-Z+€3 $O'*56$ZB1
¡LOPh$!'*)@3 )2Rˆ-1)2&E3 -?N!-0/ 'V9;/ -1B/ )@3*3 -?&g$H&49H828l3 56-[¢9;=?-?'$H[ 9=0S6x-!a($!&(-1$!&g-Z9;=M5l‘£‘3-Z9;=M54=0$H/ &6-M/<¤¢R!-0/*3 -0¥6¦%$H[Q3 5(-=?S6x-Ha
3 56-#7./ $L+(S6=M3$;[3 56-€&LS6^x-M/ '1$!&—3 56-#[o9H=?-?'3 5Y9†3^#-?-M39†3<3 5(-#=?$H/ &(-0/O)2'B/ )@3*3 -?&q–‡‘5(->'*S(^§$H[3 56-€&LS6^<x-0/ '
B/ )@3*3 -?&#9;3u9;8:8.3 56-=?$;/ &6-0/ 'J)2'%¡H¨!¨;©(Jªb[Y«+.-?&6$;3 -?'Q3 56-‘'*S(^W$H[3 5(-‘&LS6^x-M/ 'Q$!&#9;8:8L3 56-‘[o9H=?-0'?aE¬Y&Y+<9H828L3 56-7$!'*'*)2x(8:-
R!9;82S6-?'$;[Q«
jlkmonqpZr]ksut
v -03>­V®¯°#d1Uw1±ƒx-X9 =?S(x-!a9;&Y+²3 5(-X&LS6^<x-0/ 'O³e“´;e µHe*¶Ye*·Ee ¸šx-XB/ )@3*3 -?&š$H&3 56-\[¢9H=0-?'>­K®<¯°Xa®<Uw1¯<a
d1Uw1±%a­Kd±Q°Xal­K®Uda¯w1±Q°}/ -0'*7-?=03 )2Rˆ-?8@cE1‡‘5(-?&43 56-7(/ $L+.S6=03 'B/ )23*3 -0&œ9†3K3 5(-<=?$;/ &6-0/ '­al®#aq¯<al°Xalda
UOa6wa±C9;/ -1/ -?'*7-0=03 )2Rˆ-?8@c>³.¶L·ˆa(³L´?·ˆa.³.´?¸qa6³L¶¹¸qa6µM¶L·ˆa(´Mµ?·ˆa(´0µZ¸qa6µ0¶¹¸qu‡‘56-1'*S6^$H[Q3 56-0'*-1º&LS6^<x-0/ '‘)2' t
S R

D C

P Q

A B

»
‚ ¤¼·u™¸¦0¤¢³L´ ™C´0µ ™Cµ0¶K™Ž³L¶.¦Q‚`¤¼·‘™²¸¦0¤¢³1™—µZ¦0¤¢´ ™C¶.¦M½
‡‘56)2'1)2'N!)2Rˆ-?&–3 $Xx-O-ZTES69H8Q3 $D¡H¨H¨H©g‚¡ ˜V¾!¿¾ ?ÀˆÁL\~Kx('*-0/ Rˆ-3 569;3&($!&6-$H[%3 5(-[¢9;=03 $H/ '³™šµHaz´™š¶Yaz·™¸—):'
-ZTESY9;8l3 $gH‡‘5LS6'¤¢³1™²µ?¦0¤¢´ ™²¶.¦0¤¼·%™¸¦)2'V-ZTES69H8q3 $#© ¾†¿¾ ?ÀˆÁEaq¡ ¾ À ¾ ZÀˆÁEaq¡ ¾†¿¾†¿!¿ ©>$;/1¡ ¾ ¡ ¾H ¨.!‘žK-?&(=?-13 56-
7$!'*'*)2x682-RH9H82S6-?'$;[Q«Ã‚³™C´ ™Cµu™—¶™Ž·‘™²¸49;/ -1©V™ ¿ ™ZÀ!Á1‚WZÁ;©.a¡™ŽÀV™š?ÀˆÁ‚ƒ†Á  a¡™ ¿ ™ ¿H¿ ©<‚ ¿!¿!Ä a
$H/K¡™²¡™  ¨.K‚  ¨ Â
‡‘5LS6'‘3 56-M/ -9;/ -1©O7$H'*'*):x(82-RH9H82S(-?'$H[Q«y9H&6+>3 5(-0c\9;/ -O†Á†©(a2†Á  a ¿!¿HÄ a  ¨ Â
¿ v -03ÅÆ9;Ë &Y+DÇË x-3 5(-O/ $E$H3 '$H[3 5(-#TESY9!+L/9;3 )2=#-ZTESY9†3 )2$!& ’ ˜ ™šÈX’  #‚G¨.aqB56-0/ -#ÈÉ):'<9H&œ$L+6+ )2&E3 -?NH-0/Z v -03
ÊYË ‚Å ™—Ç aL[o$H/V̜ͨL%PQ/ $†Rˆ-K3 569;3[o$;/V̖ͨ.a
¤¼9ˆ¦ ÊYË )2'V9H&\):&E3 -?NH-0/ZÎ9;&Y+
¤¢xY¦ N!=Z+–¤ ÊYË e ÊYË!ÏQÐ ¦ ‚ƒH
jlkmonqpZr]ksut ›.)2&6=0-gÅÑ9H&Y+ŽÇW9†/ -g3 56-g/ $E$H3 '>$H[K3 56-4-ZTESY9†3 )2$!&’ ˜ ™ŸÈ\’  —‚ɨLaB‘-4569†R!-gÅ ˜ ™yÈ\Å  C‚
¨.e“Ë Ç ˜ ™yÈ>ËLÇ ÔzÐ  ŽË.‚ÒÔ ¨.½ÓXS(823 )27(82c.)2&6NŽxEcÅ Ë.Ô ˜ e“Ç ËLÔ ˜ / -?'*7-?=03 )2Rˆ-?8@cB‘-D5Y9ZRˆ-DÅ Ë ™yÈXÅ ËLÔzÐ  Å Ë.Ô ˜ ‚Ò¨Ž9H&6+
Ç ™ŽÈ>Ç  Ç ˜ ‚èL
£K+(+()2&6N€B‘-#$Hx(39H)2&œÅ Ë ™Ç Ë ‚  È ¤¢Å ËLÔzÐ ™Ç Ë.ÔqÐ ¦Q™ƒ¤¢Å Ë.Ô ˜™Ç ËLÔ ˜Z¦M€‡‘5()2'NH)2Rˆ-?'9\/ -?=0S(/*/ -?&(=?-/ -?8:9;3 )2$H& [o$;/
̖͚¡ t
ÊYË ‚  ÊYËLÔzÐ ™ ÊYËLÔ ˜ e*ÌDÕ¡ ¤]Ö.¦
¤¼9ˆ¦—×K$ZB ÊYØ ‚W™1‚y¡O9H&6+ ÊqÐ ‚ÆÅ4™—Ç—‚  Ȝ‘‡‘5.S(' ÊYØ 9;&Y+ ÊqÐ 9;/ -)2&E3 -?N!-0/ '?Ù cg)2&Y+.S6=03 )2$!&laY)@3[o$!8282$ZB'
[]/ $H^ ¤]Ö.¦%3 5Y9†3 ÊYË )2'9H&g)2&E3 -?N!-M/[]$H/V-Z9H=5X̖ͨ.
¤¢xY¦œ„—-9HN!9H)2&–S6'*-€¤]Ö.¦3 $>7./ $†Rˆ-xLc\)2&Y+.S6=03 )2$!&43 569;3VN!=Z+C¤ Ê Ë e Ê ËˆÏhÐ ¦‚ÑH‡‘5():'V)2'K=?82-Z9†/ 82c43*/ S(-1[o$H/Ì ‚y¨.al9H'
N!=Z+–¤b¡Le  Èg¦Q‚`HaYxEc#3 56-1NH)2Rˆ-?&X=0$!&Y+.)@3 ):$H&X3 5Y9†3ÈÚ)2'$L+6+l
v -03VN!=?+—¤ ÊYË.Ô ˜ e ÊYË.ÔzÐ ‚Ñ!e“Ì—ÍŸ¡Lªb[u)@3B‘-0/ -3 $#5Y9;767-?&g3 569;3N!=?+Ž¤ ÊYËLÔzÐ e ÊYË ¦ۃ!aY39;ˆ-9€7./ )2^#-ÜD3 569;3
+()2RL):+(-?'x$;3 5 Ê ËLÔzÐ 9H&Y+ Ê Ë ‡‘5(-?&œ[,/ $!^Ò¤]ÖL¦MaB‘-ONH-033 5Y9†3ܗ+.)2R.):+.-?' Ê Ë.Ô 9H82'*$.O‡‘5LS6'%Ü )2'9€[o9H=03 $;/$H[
N!=Z+Ž¤ Ê Ë.Ô ˜ e Ê ËLÔzÐ ¦Maq9>=0$!&E3*/9!+.):=M3 ):$H&q›.$€NH=Z+²¤ Ê ËLÔzÐ e Ê Ë ¦‚GHžK-?&(=?-B%-56˜ 9†R!-N!=?+²¤ Ê Ë e Ê ËˆÏhÐ ¦‚!a[o$;/
9H828l̜ͨ.

©(OPJ/ $†Rˆ-u3 5Y9;3Q3 5(-‘&LS6^<x-0/Q$H[63*/ ):7(Ë 82-?'¤¼­Ëe ®#e¯1Ë ¦qB56-M/ -­e ®#e¯9;/ -'*S(x6'*-03 'J$;[qÝEHeM¡Le ¾?¾?¾ e*ÌzÞ'*S(=M53 5Y9;3 ­€ß®gßV¯y‚
à e ­Žß>®_‚ á à e®ßX¯‚Ãá ¨)2'Á  ¡E½ À ™ Â
jlkmonqpZr]ksut
A
B
2 3
1

5
4

6 C
7
X

v -03>|â‚ÒÝE!eM¡Ee ¿ e ¾0¾?¾ e“Ìhވ`„—-4S('*-4ãu-0&6&+.):9HNH/9;^ä[]$H/\'*-03 '\­e ®#e¯Ú3 $C'*$!82Rˆ-43 56-47./ $!x(8:-0^g`‡‘5(-D/ -0N!)2$!&('


$H3 5(-0/K3 569H&–­šßg®Ÿß4¯i¤uB56)2=5œ)2'K3 $>x--?^#7(3fcY¦9†/ -&LS6^<x-0/ -Z+ !a ¡Ea ¿ a ©(a  a À.a Á49H'K'*56$ZB&œ)2&43 56-¬YNHS(/ -!Îl-! N.:a 
=?$H/*/ -0'*7$!&Y+.'3 $g­å.¤¼®y斯1¦  ­šß–®ç斯Kç;aJ¡\=?$H/*/ -0'*7$!&Y+.'3 $g­Ãߖ®<å!¯i‚G­šß–®ƒßœ¯Kç†eJÁ\=0$H/*/ -?'*7$H&Y+('3 $
|Då.¤¼­Žæ>®ÃæX¯1¦ ‚íVçhßX®çhßg¯Kç†a6'*)2&6=0-­Žß\®Ãß\¯Ÿ‚ à
è
Œ Ë )@/ '“3 82c43 5(-&LS6^<x-0/V$H[%B‘9?c('K$H[9;'*'*)2N!&6)2&(Ng-?82-?^#-?&E3 '$;[%|é3 $€3 56-<&LS6^x-M/ '/ -?NH)2$!&6'B)@3 5($!S(39;&Ec4=?$!&6+()@3 )2$!&œ):'
Á Ž£V^#$H&6Ng3 5(-?'*->3 5(-0/ -\9;/ -X=?9H'*-?'#)2&CB56)2=5š¡4$H/  $;/Ox$;3 59†/ -X-?^#7.3¹cEœ‡‘56->&LS6^<x-0/$;[+.)2'“3*/ ):x(S(3 )2$!&(')2&
B56)2=5C¡>)2'1-?^#7.3¹c4)2'À Ë v ):!-0B)2'*-O3 56-&LS(^x-0/K$H[+.):'“3*/ )2x(S(3 )2$!&('1):&–B56)2=5  )2'-?^#7.3¹cg)2'9H82'*$4À Ë OÙ%S(313 5(-?&
B‘-O569ZRˆ-<'*S(x(3*/9;=03 -Z+–3fB)2=?-3 56-<&LS6^x-M/$H[‘+()2'“3*/ )2x6S.3 ):$H&6'1)2&DB56)2=M5œx$H3 5D3 56-/ -?N!)2$!&('<¡\9H&6+  9;/ -O-0^#7(3¹cE›.Ë $
3 $O=?$H^#7-?&6' 9†3 -B%-5Y9ZRˆ-3 $€9!+(+\3 56-1&LS(^x-0/$;[ +()2'“3*/ Ë )2x6S(3 )2Ë$H&6'):&gË B5()2Ë=M54x$;Ë3 5D¡O9H&6Ë +  9;Ë / --?^#7.3¹cE%‡‘56)2'V)2' Â
žK-?&(=?-K3 5(-1+(-?'*)@/ -Z+\&.S(^x-0/‘$H[h3*/ )27682-?'¤¼­e ®€e ¯1¦)2&gÁ  À  À ™  ‚ÆÁ  ¡E½ À ™ Â
 v -03 ­V®¯°`x-‘9KTESY9H+./ )28:9;3 -0/9;8¼Îˆ’#9H&6+ŠŸx-%3 56-%^#),+.7$!)2&E3 'h$H[­¯š9;&Y+®°ƒ/ -?'*7-?=M3 ):R!-?8@cO9;&Y+13 56-‘82)2&6-?'Q3 5./ $!S6NH5
|_9H&Y+OŠy/ -?'*7-?=03 )2Rˆ-?8@c7Y9;/9;828:-08Y3 $<®°Xe ­K¯Æ^#-?-M3%)2&€F v -M3‘d‘eUOe*w<e ±–x-3 56-^#):+(7$!)2&E3 'J$H[q­K®#e ®<¯Ke¯°Xe °#­
/ -?'*7-?=03 )2Rˆ-082cE PQ/ $†Rˆ-3 5Y9†3
¤¼9ˆ¦CTESY9!+L/ )28,9†3 -0/9H82'V­Kd1F±C9H&6+g­Kd|4±C569ZRˆ-K3 56-K' 9H^#-9;/ -?9.Î
¤¢xY¦œ3 56-19†/ -Z9H'$H[z3 56-TESY9H+./ )28:9;3 -0/9;8z­Vd1F±Qe ®UF1d‘e¯wF1UOe °€±QF1w‹9;/ -9H828q-ZTESY9H8¢
jlkmonqpZr]ksut
„—-1S('*-3 5(-1[¢9H=M3 ' t ¤¢),¦3 5(-82)2&6-ê¹$!)2&6)2&6N#3 5(-^#):+.7$!)2&E3 '$;[J3 56-1'*):+(-?'V$H[u93*/ ):9H&(N!82-<)2'V7Y9;/9;828:-08z3 $O3 56-13 56)@/+g'*):+(-HÎ
¤¢)2)]¦9;&Lc\^#-Z+.),9;&X$;[ 93*/ ),9;&6NH8:-x():'*-0=03 'V)@3 '9;/ -?9.ÎQ¤¢)2)2),¦3fB‘$O3*/ ):9H&(N!82-?'5Y9ZRL):&(NO-ZTESY9;8qx69H'*-?'V9H&6+Xx$!S(&Y+.-Z+\xEc>' 9H^#-
7Y9†/9H8282-?8z82)2&6-?'569†R!--ZTESY9;8z9†/ -Z9.
C

D
X
Q

O
S
Y

A P B

¼¤ 9ˆ¦—×K$ZBš®<°‹)2'%7Y9†/9H8282-?8Y3 $d±œ9;'uB%-?8289H'%F| u›.$F|_)2'%7Y9†/9H8282-?863 $d±%QžK-0&6=?-1ë d|D±zìl‚`ë d1F±zìb £V+6+.):&(N


ë d1­±zì3 $Ox$H3 5\'*),+.-?'B‘-1N!-M3<ë ­Vd|D±zì‚`ë ­Kd1F±zì¼%‡‘5()2'7./ $†Rˆ-?'7Y9†/*3¤¼9ˆ¦M
¤¢xY¦ ×K$ZB
ë ­Kd|D±zìí‚ ë  ­Kd|XìE™Æë ­ ±h|Xì
‚ ¡ ë ­K®|Xì.™ ¡ ë ­V°O|Xì‚ © ë ­K®<¯VìL™ © ë ­V°€¯Vì
‚ © ë ­K®<¯°ì¼½
žK-?&(=?->xLc¤¼9ˆ¦Maë ­Vd1F±hÐ ì‚ îÐ ë ­K®<¯°ì¼›L)2^#):8:9†/ 82cŽxEcC'“c.^#^#-03*/*cœ-Z9;=M5²$H[V3 56-g9†/ -Z9H'4ë ­VUF1dKì¼eZë ¯wF1UìV9H&6+
ë °€±JF1wì)2'-ZTESY9;8u3 $ î ë ­K®<¯°ì¼ ‡‘5LS('3 5(-€[]$!S./ON!)2Rˆ-0&Ž9;/ -Z9;'O9;/ ->-ZTESY9;8¢–‡‘56)2'7./ $;R!-?'<769;/*3\¤¢x¦MŽë ×K$;3 - t ëqì
+(-?&($H3 -?'9†/ -Z9;ì¼
À. v -03Oï@Ü Ð e¢Ü ˜ e¢ÜðHe ¾0¾Z¾ e¢Ü Ë e ¾0¾?¾ñ x-9>'*-ZTES(-?&6=?-<$H[%7(/ )2^#-?'1+.-0¬Y&(-Z+4xLcOÜ Ð ‚¡€9H&6+D[]$H/ÌÍÑ!eoÜ ËˆÏhÐ )2'3 56-<8:9;/ N!-0'“3
7(/ )2^#-V[o9H=03 $;/K$;[Ü Ð Ü ˜ ¾?¾?¾ Ü Ë ™!½q¤¼‡‘5.S('JÜ ˜ ‚ ¿ e¼Üð‚ÆÁ!¦MuPQ/ $†Rˆ-3 5Y9†3JÜ Ë ‚ á  [o$;/V9H&Ec>ÌJ
jlkmonqpZr]ksut ÙucÃ+(9;39gÜ Ð ‚ò¡Ee¢Ü ‚ ¿ e¢Ü ð ‚éÁL‹ªf3€[]$!8282$†B'XxLc)2&Y+.S6=03 )2$!&3 5Y9;3<Ü Ë e*ÌóÍô¡—)2'\$L+(+lGë Œ $;/g)@[
Ü ˜ e¢ÜYðHe ¾?¾?¾ e¢Ü ËLÔzÐ 9†/ -$L+6+a.3 5(-?&Ü ˜ Ð Ü ˜ ¾Z¾?¾ Ü ËLÔzÐ ™K):'9;82'*$#$L+6+>9H&Y+>&6$;/ ¿ ‡‘56)2'9H82'*$[]$!8282$ZB'VxEc€)2&Y+.S6=03 )2$!&l Œ $H/
)@[ÜðV‚yÁ9;&Y+X)@[Üð;e¢Ü e ¾0¾?¾ Ü Ë.ÔqÐ 9;/ -&6-0)23 5(-0/K¡<&6$;/ ¿ a63 5(-?&OÜ Ð Ü ˜ Üð ¾?¾?¾ Ü Ë.ÔzÐ ™9;/ -1&6-0)23 5(-0/xEcg¡&($H/xEc ¿ ‘›.$
Ü Ë ):'&(-?)@3 56-0/K¡<&($H/ ¿ î
õ
  !"$#&%')(**+
,.-/ 021346587:9;0=<?>2@BADC.02EGFIHJ<BKLNMPOP<)1N0QLR<BOTSVUXWZYXWZ[6WJ\]9;081N^0`_XMaKVb;@BMPA.1dce@)f34GWZ4g5gWZ587!WJ7h3iLN0kj
cNb;0Q>l1RMmC.0kOPn6cNHJ>d^61N^J<)11NLRMP<BAoBOP0Qc3Y`[p<)AZKX5`\Uq<BLN0r0QFIHMPOP<)1N0QLR<BOT-tsLR@uCB0v1N^J<)1346587Mac<8LR^@B_69HJcQ-
w 021R0kLR_XMmA0`Mm1Rc<)Ao.Om0xck-
y z;{}|~Q€}z;v‚ƒ 0!^J<„CB0!Y`[†‡4ˆ7Š‰B‹ŒU=\-ŽDMPAJ>k03Y`[<BAJK$5`\U‘<BLN0’9;@)1R^“0xFIHMmOa<u1R0kLd<)O<BAJK
Y=[”$U=\eW.1R^0knX_6HJc 19;0>k@BAo.LNH0QAI11NLRMP<BAoBOP0QcQ-–•^MPcvMP_gbJOmMP0Qcv1N^Z<u1e3Y—˜Y`[i”[83™˜5`\Œ˜\U
—U=5g-šOacN@h›58œˆ˜ŽžBŸIq›r[`Y`3ˆ-¡¢D0k0=£tMPoJ-e,.- ¤
D R C

V F
L
W Q
S
M O
U

A P B

Fig. 1.
¥ 0kAJ>k05`\¦MPcGbJ<)Ld<)OPOP0kO1N@ŒY=3=-˜§@u¨©5`\ª«Y`3:MP_XbOPMm0xc?1N^Z<u1Š5`\Y`3MPch<bZ<)Ld<)OPOm0QOm@.oBLd<)_’-$¬­A
bZ<)LN1NMa>2HOa<)L=\3Mac8bJ<)Ld<)OPOP0kOt1N@5`Yª<)AJK®\3¯5`YX-?•^Mac`c^@u¨c1R^J<u1ˆ37‡Mac8bJ<BLR<BOmOP0kOt1N@!4g5°<BAJK
37°$4g5g- ¥ 0QAJ>20`34g587:MPce<gbJ<BLR<BOmOP0kOP@Bo.LR<B_’-
/ 0211R^0KVMa<)o.@BAJ<BO35”<)AZK?4ˆ7±9Mac0x>l10x<B>d^X@)1R^0kL<)1v²±-•^0kA?7G²³$7G4g‰)‹`qY`[qq5`\®p3[6-
•^DHJc=MmA1RLNMa<)Ao.Om0?37G5gW 1R^0h_X0QKMP<BAJc=3[gWt7G²±W–5`\”<BLN0?<)OPO0xF.HZ<)O¢-G•^DHJc=37Š5°Mac`0xF.HJMmOa<u1R0kLd<)O¢-
•^JMPceMP_XbOPMm0xc34g587ªMac<gLR^@B_69HJcQ-–´@BLR0k@uCB0QL1R^0=<)AJoBOP0Qc<)LR0`žBŸ.8<BAJK,x‹)ŸIu-
‹-r¬µf·¶¸¹?<)LR0eMPA.1R0ko.0kLdckWI<)AJK’,uº8KVMPCIMaKV0xc9;@)1N^h1N^002EVbLR0QcRcMP@BAJc–¶;»–¼½‹)¶J¹v¾’¹V»t¼½¿)¶¾Àº„¹g<)AJKX¶;»–¼ŠÁB¶J¹v¾
‹)¹V»¾®¶h¼Â¹ÃWV1N^J0kA’bLR@uCB01N^J<)18,uº6KVMmCDMaKV0Qc¶J¹ˆ¼“,x‹)¶6¾$,„¿u¹Ã-
y z;{}|~Q€}z;v‚Ä 9JcN0kLRCB01N^J<)1¶;»¼ÀÁ)¶Z¹¾Å‹)¹D»¾Â¶X¼¹?i¡}¶g¼¹¤2¡Æ¶X¼‹)¹¾Ç,„¤l-•^DHJc,„ºˆKVMmCDMaKV0Qc0QMÈ1R^0kL
¶G¼¹Š@BL¶h¼®‹u¹8¾p,B-DHJbbZ@Ic081R^J<u1=,uº6KVMmCDMaKV0Qc¶G¼¹Ã-¬­AŠ1N^Mac>k<.c08¶½É$¹Ž¡Æ_X@VKÂ,„ºB¤v<BAJKŠ^0kAJ>k0
¶ » ¼‹)¶J¹`¾®¹ » ¼®¿u¶g¾“º„¹gÉp¹ » ¼‹)¹ » ¾®¹ » ¼®¿u¹8¾“º„¹XɎ‹)¹ ¡Æ_X@VKÂ,„º)¤2Ê
•^DHJcr1R^0`oBMPCB0QAŠ>k@BAJKVMm1NMP@BAŠ1N^Z<u1=,„ºˆKVMPCDMPKV0xce¶Z»¼®‹u¶Z¹¾¹V»¼®¿u¶ˆ¾“º„¹?MP_gbJOmMP0Qc1R^J<u1`,uºˆ<BOPcN@XKVMmCDMaKV0Qc
‹)¹Š<BAJK’^0kAZ>20`¹½MÈ1dc0QOÈf -ËrHV11N^J0kA!¶Ép¹q¡Æ_X@VK,uº)¤rMP_XbOmMP0Qc1N^J<)1=,„ºgKVMPCDMPK0Qc¶<)Oac@Z- ¥ 0QAJ>20ˆMmA’1R^MPc
>Q<BcN0X,„º6KVMPCIMaKV0xc¶J¹g¼],„‹u¶6¾p,x¿u¹Ã-
VHbb;@.cN0`@BAŠ1N^0`@B1N^0QLe^J<)AJKŠ1R^J<u1=,uº=KVMPCDMPK0Qce¶h¼®‹u¹8¾$,.-•^IHZc¶½Ép‹)¹ˆ¼“,¡}_g@VK,uº)¤r<)AJKŠ^0QAJ>20
¶ » ¼®‹u¶Z¹8¾Å¹ » ¼¿)¶g¾]ºu¹gÉǹ » ¼®¿u¹`¾Åž ¡}_g@VKÂ,„ºB¤lÊ
•^DHJc=,„ºˆKVMmCDMaKV0Qc¹D»¼®¿u¹`¾ÅžJ-ËvH1¶½Ép‹)¹ˆ¼“,¡}_g@VK,uº)¤r<)Oac@gMP_gbJOmMP0Qc1R^J<u1
¶J¹g¼],„‹u¶6¾p,x¿u¹XÉp‹·ÌT¹ » ¼¿)¹8¾Œž.ÍΡ}_g@VKÂ,„ºB¤lÊ
VMmAJ>k0?,„º6KVMPCDMPKV0xc¹V»e¼¿)¹¾ŒžWMm1efÆ@BOPOm@u¨cv1R^J<u1=,uºˆKMmCDMaKV0Qc¶Z¹6¼“,x‹)¶6¾$,„¿u¹Ã-
ÁJ-r¬µfrÏZ¸dЄ¸Nш<)LR0=1N^JLN0Q0ˆLR0Q<)OtADH_ˆ9;0kLdccHJ>d^1N^J<)1?Ò ÏX¼ŒÐ)Ò;Ó¯Ò ÑBÒÈWÒ Ð¼ÅÑBÒÃÓÒ ÏÔÒÈWÒ Ñe¼ÅÏÔÒÃÓ¯Ò Ð)ÒÕW;1R^0kAbLN@uC.0
1R^J<u1@.A08@)fÏ;¸RЄ¸RÑMacr1N^0=cNH_ª@)ft1N^0`@B1N^0QL1 ¨v@Z-
y z;{}|~Q€}z;v‚tÖ cMPAogÒ Ï·¼`Ð)Ò.Ó™Ò ÑBÒÈW„¨v0v@B9V1d<)MPAh¡}Ï ¼8Ð2¤»ӓÑ2»¨e^MP>d^ˆMPc0QFIHMPCu<)OP0kAI1·1N@g¡}Ï ¼8Ð.¼8Ñk¤2¡TÏ·¼`Є¾ˆÑk¤Ó
ŸJ-tDMP_XMmOa<)LROPnBW¡TÐV¼=ÑV¼=ÏV¤2¡TÐV¼=Ñ.¾gÏV¤tӓŸ<)AJKh¡TÑV¼=Ït¼ˆÐ2¤2¡TÑV¼=Ï·¾XÐ2¤tÓǟ-´HOÈ1RMmbJOmnDMPAo1R^0QcN0rMmA0xFIHJ<)OPMÈ1RMm0xckW
¨r0`oB0k1
¼=¡}Ͼ“Ðv¼Ñk¤ » ¡TоÅѼÏV¤ » ¡}ѾŒÏ=¼®Ð2¤ » ӓŸÊ
•^JMPc8fÆ@.LR>k0Qc1N^Z<u1 {}×tØ MPc80xFIHJ<)O–1R@½ÙQ0kLR@J- ¥ 0QAJ>20?Mm18fÆ@BOPOm@u¨c1R^J<u1`0kMm1N^J0kL=Ï6¾$Ðg±Ñˆ@.LˆÐv¾ÇÑX±Ï@.L
ÑŽÏgŽÐ„-
Ú -£tMPAJK’1R^0ˆADH_69Z0QL@Bf<BOmO¿„j­KVMPoBMm1ADH_ˆ9;0kLdcˆ¡}MmAÀ9J<BcN0h,QŸI¤0Q<.>d^!@)f¨e^Ma>d^À>2@.AI1R<)MPAJc1R^0ˆ9JOm@V>dÛÂ,x¿?<BAJK
MacKVMPCDMPcNMP9Om089DnÀ,„¿V-e¡}£J@BLe0kEV<B_XbOm0.WÁ,„¿ Ú ¿WIÁ Ú ,„¿B¿6<)LR01 ¨r@?cHJ>d^½ADH_ˆ9;0kLdcQ- ¤
y z;{}|~Q€}z;v‚ šADnŠcNHJ>d^’AIHJ_ˆ9;0kLcN^@BHJOPKŠ9Z0ˆ9Z@B1N^KVMPCDMPcNMm9JOm0`9Dn½¿X<BAJK’Á-•^J0`OP<.c 1KVMPoBMm1@)f<XADH_ˆ9;0kL
KMmCDMacMP9OP0?9In¿½_ˆHJc1=9;0?0kMm1N^0QL6¿½@BL=ŸJ- ¥ 0kAZ>20G<BAIncHJ>d^ADH_69Z0QL8f}<)OPOPc`MPAI1N@!@.A0?@)fv1N^0?fÆ@.OmOP@u¨eMPAo
cN0kC.0kA’>k<)1N0ko.@BLRMm0xckÜ
¡}Ma¤ÏDÐ2Ñu,x¿S ¡ÆMPMݤÏDÐu,x¿)ŸJS ¡}MmMPMa¤ÏVЄ,„¿B¿S ¡ÆMPC¤Ï;,x¿BÐ2ŸS ¡}C¤ÏZ,„¿BÐ2¿SÔ¡ÆCDMa¤,„¿)ÏVÐlŸJS ¡ÆCDMPMa¤,„¿)ÏVÐ2¿-
¥ 0kLR0=Ï;¸RЄ¸RÑ<)LR0`KVMPoBMm1RcQ-/·021HZce>2@BHJA.1e^@u¨q_?<)ADnhADH_ˆ9;0kLdc@)f–0Q<.>d^½>Q<u1R0koB@.LNn?<BLN01N^J0kLR0B-
¡}Ma¤g¬­A]1N^JMPc?>Q<BcN0’Ïi‡Ÿ Þ Wv<BAJK®1R^0Áuj­KVMPoBMm1gADH_69Z0QLhÏVÐlÑhMac?KVMPCIMacNMm9OP0’9DnŒÁWr<)AJK]^0kAZ>20’@BA0½@)f1N^0
ADH_69Z0QLRcMPA½1N^0=cN021`ßI,QŸI‹V¸k,xŸ.¿¸kÊkÊQÊJ¸RàBàBàá.-–•^MacoBMPCB0QceÁ.ŸBŸ6ADH_ˆ9;0kLdck-
¡}MmMݤršoI<)MPAŠ<6AIHJ_ˆ9;0kL@)f·1R^0fÆ@BLR_âÏVЄ,x¿BŸ`MPcKVMPCDMPcNMP9Om09Dn!,x¿ˆMÈft<)AJKG@BAOPn?MÈf1N^0=‹ujµKVMPoBMm1vADH_69Z0QLeÏDÐMPc
KMmCDMacMP9OP0`9InŠÁ- ¥ 0QAJ>208Mm1_ˆHJc19;08fÆLN@._¯1N^0=cN021`ßI,x‹¸k,x¿¸kÊQÊkÊ;¸NàBàá.-–•^0kLR0`<)LR0`ÁBŸXcHZ>d^ŠADH_69Z0QLRcQ-
¡}MmMPMݤešcMPA®¡ÆMPMa¤2WJ^0QLN0`<)LR08<)o.<BMmA’ÁBŸgADH_69Z0QLRcQ-
¡}MmC¤eDMP_XMmOa<)L1N@!¡ÆMPMa¤2SZÁ.ŸˆADH_69Z0QLRcQ-
¡}C¤eDMm_XMPOP<BLe1N@!¡}MmMݤlWJÁ.ŸgAIHJ_ˆ9;0kLdck-
¡}CDMa¤ ƒ 0ˆ>Q<)A’9Z0QoBMPA½1N^0ˆ<BAJ<)OPnVcMac@)ft1N^J0=ADH_ˆ9;0kL@Bf1R^0`fÆ@.LN_‡,x¿)ÏVÐlŸg<BcMmAÅ¡}MmMݤl- ¥ 0QLN0=<Bo.<)MPA’ÏDÐ<.ce<
‹ujµKMmo.MÈ1AIHJ_ˆ9;0kL8_ˆHJc189Z0XKVMPCIMacNMm9OP0g9In!ÁJW;9JHV18Ï!™ŸŠMac<BOPcN@Gb;0kLR_XMPcRcMP9OP0B- ¥ 0QAJ>20gMm1_6HJc 189;0ˆfÆLR@B_
1R^0=cN021`ßxŸ.Ÿ¸RŸBÁ¸RŸBžJ¸kÊkÊQÊJ¸RàBàVáI-t•^J0kLR08<BLN0`Á Ú cNHJ>d^’ADH_ˆ9;0kLdck-
¡}CDMmMݤ ¥ 0QLN0=<Bo.<)MPAh1R^0kLR0=<)LR08ÁBÁgADH_ˆ9;0kLdckSÏVÐ_ˆHJc19;08fÆLN@._¯1N^0=cN021`߄Ÿ,.¸NŸ Ú ¸NŸDºD¸kÊQÊkÊZ¸NàIºDá.-
šKKVMPAoh<)OPOÃ1N^J0QcN0`¨v08oB0k1ÁBŸ.Ÿe¾ŒÁBŸ¾ŒÁBŸ¾ŒÁBŸ¾ŒÁBŸ¾ŒÁ Ú ¾ÅÁ.Á= ÚIã º=AIHJ_ˆ9;0kLdck-
¥ @u¨v0QCB0QL`1N^Mac6MPcˆA@)161N^0½>2@BLRLR0Q>l1`äZoBHLR0G<Bc=1N^0QLN0GMPc6@uCB0QL=>2@BHJA.1RMmAJoJ-À/ 0216HZcˆcN0k0h^@u¨_6HJ>d^Å@uCB0QL
>k@BHAI1RMmAoMPcgKV@.A0G9DnOm@D@.ÛIMPAoÀ<)1ˆ1R^0GMPAI1N0kLdcN0Q>l1RMm@.AÅ@)f0x<B>d^®bJ<BMmLg@)f>Q<u1R0koB@.LNMP0QcQ-šªAIHJ_ˆ9;0kLgMmAŽ¡}Ma¤
@.9DCIMP@BHZcOPn>k<BAA@)1`OPMm0?MPA$¡ÆMPMa¤2Wv¡}MmC¤@BLh¡}CDMa¤8<.c8MPc80QCIMaKV0QA.18fÆLR@B_‘1N^0?Oa<Bc1=KVMmo.MÈ1x-?•^0kLR0?>k<)AJA@)1`9;0h<
>k@B_X_X@BA½ADH_ˆ9;0kLMPA®¡ÆMݤ<)AJK¡}MmMPMa¤<.c<)ADnX1 ¨r@?cHJ>d^’ADH_69Z0QLRceKVMmåÃ0kLMPA½1N^0 Ú jT1R^½KVMPoBMm1Q-¬µf–<gADH_ˆ9;0kL
9;0kOP@BAJo.cv1N@g9Z@B1N^¡ÆMݤv<BAJKÀ¡}C¤lWD1N^J0kAŠcNHJ>d^½<ˆADH_69Z0QLv@Bf·1N^0fÆ@.LN_¯Ï;,x¿,x¿V-–•^MacrMPcKVMPCDMPcNMm9JOm09DnhÁ=@BAJOmn
fÆ@.LˆÏâÁ¸NžJ¸NàJ-h•^IHZc81N^0QLN0h<BLN0?Á’>2@._X_g@.AAIHJ_ˆ9;0kLdc`MmA$¡}Ma¤`<BAJKÇ¡ÆMPMa¤2-ŠšADH_69Z0QL=¨e^Ma>d^ÂMac`9;@)1R^
MPAp¡ÆMݤ8<)AZK“¡}CDMmMݤ8Mac`@)fr1N^0hfÆ@BLR_†,x¿BÑu,x¿Š<)AJKKVMPCDMPcNMP9MmOPMm1 nÀ9DnÁ’o.MmC.0Qc`Ñ?¯Ÿ¸RÁ¸Rž¸NàJSÔ1N^DHJcˆ¨v0X^J<„C.0 Ú
ADH_69Z0QLRce>k@B_X_X@BA½MmA®¡}Ma¤e<BAJK¡ÆCDMmMݤl-v•^J<u1e02EV^J<BHJc1e<BOmO bZ@IcNcNMm9JMmOPMÈ1RMm0xc¨eMÈ1R^®¡ÆMݤl-
§@u¨—¡ÆMPMݤ>k<)AŠ^J<„CB0>2@._g_X@.AŠADH_69Z0QLRcr¨eMm1N^½@BAOPnh>k<)1N0ko.@BLRMm0xc¡}MmC¤r<)AZKÀ¡ÆCDMݤl-•^0QLN08<)LR0A@6ADH_69Z0QLRc
>k@B_X_X@BA!9;021 ¨r0k0QAÅ¡ÆMPMa¤<)AJK®¡}CIMݤe<.c0kCDMaKV0kAI1fÆLR@B_:ÁujµLdK½KMmo.MÈ1x-•^J0kLR0=MPc@BAOPn½@BA0`ADH_69Z0QL>2@._X_g@.A
1R@!¡ÆMPMa¤r<)AZKÀ¡ÆCDMݤlWVAJ<B_X0kOPn!,x¿V,„¿)Ÿ=<BAJKG1N^MacrMPceKVMPCDMPcNMP9Om09DnhÁ-•^0QLN0MPcrA@B1N^MPAo?>2@._X_g@.A?1N@!¡}MmMPMa¤r<BAJK
¡}C¤8<Bc8>k<BAÀ9;0XcN0k0QAÀfÆLR@B_:1R^0?ÁujµLRKKVMPoBMm1Q-6•^J0g@.AOPnADH_ˆ9;0kL`>2@._X_g@.A1R@¡}MmMPMݤ8<)AJK]¡ÆCDMmMݤMac6,„¿V,„¿B¿
<BAJKŠ1N^MacMaceA@)1KVMPCDMPcNMP9Om0`9DnGÁJ-–¬µ1>k<)A½0Q<.cMPOPnh9Z08MPAVfÆ0kLRLR0QKG1N^J<)1A@XAIHJ_ˆ9;0kLeMac>2@._g_X@.AG1R@’¡}MmC¤<BAJK
¡}CDMa¤v9DnXOm@D@BÛDMPAo6<)1v1R^0`‹„jµAJKGKVMPoBMm1Q-DMP_XMmOa<)LROPngAJ@6ADH_69Z0QLrMacr>2@B_X_X@BAG1N@½¡ÆC¤r<)AJK¡ÆCDMmMݤl-•^DHJcv1N^J0kLR0
<BLN0`ÁD¾ Ú ¾6,„ ã AIHJ_ˆ9;0kLdc¨e^MP>d^!<BLN0`>2@.HAI1N0xKh1 ¨eMa>20.-
‹
ƒ ˆ0 >k@BAJ>kOmHJK081N^J<)11N^0ˆAIHJ_ˆ9;0kL@Bf¿ujµKVMPoBMm1ADH_ˆ9;0kLdce¨e^Ma>d^!>2@.A.1d<)MPA½1N^0=9JOm@V>dÛÀ,„¿g<)AJK!KVMPCDMPcNMP9Om0`9Dn
,„¿6MPc ÚIã º¼ ã Ú º)à-
¿-r¬­Ag1RLNMa<)Ao.Om034g5gW.OP0217¦9;01N^0e_XMaKVbZ@.MmAI1@Bf;465g-–¬µf ›r37h4— Ú ¿ <)AJK?›r3587°pÁBŸ W.KV0k1N0kLR_XMmAJ0
›r4637!-
y z;{}|~Q€}z;v‚ w LR<„¨”4ˆæÇb;0kLRbZ0QAJKVMa>2HOa<)L1R@X35±<)AJKˆç @BMPA!æŒ1R@?7!-¡TD0Q0=£tMmoZ-‹V-è¤
A

15

60 45 30

B D C

Fig. 2.

VMmAJ>k0›r4658挎ÁBŸ.)W)¨r0vo.021v›584ˆæŒŽžBŸIu-DMmAZ>20e4ˆær5ÇMPc<LRMmo.^.1NjT1RLNMa<)AJoBOP0¨eMm1N^h›r4658挎ÁBŸ.)W)¨r0
^Z<„CB046殎4658‰)‹`Ž4ˆ7!-t•^DHJc MPA81NLRMa<)Ao.Om046æ7!W„¨r0@B9Zc0QLNC.0–1N^J<)1t4ˆæŎ4ˆ7i<BAJKˆ›r7h46æŒ$žBŸI)-
•^JMPcMP_gbJOmMP0Qc1R^J<u1`4ˆæv7Mac<BAÀ0xF.HJMmOa<u1R0kLd<)O1NLRMP<BAoBOP06<BAJK^0QAJ>20Xæ4éiæv7!- Ö cNMmAJo!›ræv7h4ª¦žBŸI
<BAJK›r37h4± Ú ¿B)W¨v0`o.021›r37hæ]i,x¿B)-vËrHV1›r7h3æ]i,x¿B)-v•^DHJceæv7°pæv3=- ƒ 0=^0QAJ>20`^J<„C.0
æv7âpæv3$Žæ4G-•^MacMm_XbOPMm0xc1N^Z<u1æ“MPce1N^0`>2MPLd>2H_?>20QAI1NLR0@Bf1R^081NLRMa<)Ao.Om084ˆ7h3ˆ-•^DHJc
›r4ˆ37° ‹, ›r4ˆæ7¯ ‹Â, ê ž.Ÿ ŽÁBŸ Ê
žJ- w 021R0kLR_XMmA0=<BOmO·1NLRMmbOP0Qc`¡TÏ;¸RЄ¸RÑk¤v@)f–bZ@IcMm1NMPCB08MPA.1R0ko.0kLdcecHJ>d^Š1N^Z<u1ÏhëÇÐë“Ñ<)AJK
Ï8¾ŒÐ¾ŒÑ¾ŒÏVоŒÐlѾŒÑ2ÏgŽÏVÐlѾ$,.Ê
y z;{}|~Q€}z;v‚ sHV1N1NMPAoˆÏ¼,®ì WVÐÔ¼,Ží8<)AJK?Ñ ¼,ÇîDWI1N^J00QFIHJ<)1NMP@BAX_?<„nˆ9;0¨eLRMÈ1N1N0QA?MmA?1R^0efÆ@.LN_
ìZí„î=p‹J¡Õì6¾Œíe¾Åî)¤ ¾ Ú ¸
¨e^J0kLR0vì¸NíD¸î6<)LR0MPAI1N0ko.0kLdcvcNHJ>d^G1N^J<)1eŸXëì!ë“í6ëŒîD- Ä 9Zc0QLNC.01N^Z<u1–ìŠpŸ6MPcrA@B1ebZ@IcNcNMm9JOm0.W.fÆ@.Lr1N^0QA
Ÿh¦‹¡ÈìX¾]í)¤t¾ Ú ¨e^Ma>d^MPcMP_gb;@.cRcNMm9OP0gMmAAJ@BAA0Qo.<)1NMPCB0=MPAI1N0ko.0kLdck-•^DHJc¨r0ˆ_?<„n’¨eLRMÈ1R0=1N^JMPcMPA!1N^0
fÆ@.LN_
‹ï ì;, í ¾ í„, î ¾ îd, ìvð ¾ ì;Úí„î i,BÊ
¬µfÔìÓ$ÁJWV1N^0QAíXÓ$Á?<)AJK½îXÓ$Á-v•^J0kA’Om0kfÝ1cNMPKV0ˆMPce9;@BHJAJKV0QK’9Dn½žI‰uà¾ Ú ‰B‹.º=¨e^Ma>d^½MacOP0QcRc1R^J<)A,B-
ƒ 0=>2@.AJ>2OPHJKV081R^J<u1ìŠi,8@BL‹V-
ñ6òØxó“ôZõ VHbb;@.cN0ìÀ,.-•^0kA¨v0ˆ^J<„CB0ˆí„î?i‹J¡}í¾Œî)¤t¾“žG@BL6¡Tí8¼Œ‹B¤k¡Æî`¼Å‹B¤e,xŸ-•^JMPcoBMPCB0xc
ív¼!‹=˜,BWIîr¼’‹`i,QŸ`@.Lvív¼’‹`q‹=<)AZKXîv¼!‹`Ž¿J¡ÆLR0Q>k<BOmO;íˆë]î)¤l-•^MacMP_XbOPMm0xc¡Èì ¸RíD¸î)¤™¡,B¸RÁ¸k,„‹B¤2W
¡,B¸ Ú ¸dºB¤l-
ñ6òØxó“ö·õ ¬µf–ìÅ°‹VW 1N^0?0xF.HZ<u1NMP@BALR0QKVHZ>20Qc1R@À‹)í„î½°‹¡¢‹¾pí¾Çî)¤¾ Ú @.L=í„î’âí¾Çî8¾ Ú -Š•^MPc
LR0QKHJ>20xc1N@À¡Tí¼“,„¤2¡}î¼Ç,„¤˜¿V- ¥ 0QAJ>20=í8¼“,8—,ˆ<BAJK½î¼Ç,˜¿gMac1R^0=@BAJOmn’c@.OmH1NMP@BA·-r•^MPcoBMPCB0xc
¡Èì ¸RíD¸î)¤—¡T‹V¸d‹V¸Rž.¤l-
÷ 0kCB0QL1RMmAJog9Z<B>dÛ?1R@XÏ;¸dЄ¸NÑuWV¨r08oB0k1e1N^LR0k01NLRMPbOm0xckÜ¡TÏ;¸RЄ¸RÑk¤™¡¢‹V¸ Ú ¸k,xÁ.¤lWÔ¡¢‹V¸R¿¸ ã ¤lW·¡}ÁJ¸NÁJ¸dº)¤2-
Á
ºV-/ 021Ï;¸RÐu¸NÑ9Z081R^LN0Q08bZ@IcMm1NMPCB0`LR0Q<BO;ADH_69Z0QLRcecNHJ>d^½1N^Z<u1Ï8¾]оÅÑi,B-/·0k1
ø p_gMPAˆù.ÏIúr¾ÅÏ » Ð2Ñu¸Ðlú¾ÅÏVÐ » Ñu¸Ñ2ú¾ŒÏVÐlÑ ».û Ê
sLN@uC.01R^J<u11R^0`LN@D@B1Rc@)ft1N^J0`0QFIHJ<u1RMm@.AŠ¶;»v¾Å¶g¾ Ú ø $ŸX<BLN08LR0Q<)O¢-
y z;{}|~Q€}z;v‚ DHbb;@.cN0`1N^J0ˆ0QFIHJ<)1NMP@BA’¶Z»¾Œ¶X¾ Ú ø ”ŸX^J<.cA@hLR0Q<BO·LR@I@B1RcQ-r•^J0kA®,¼Ç,Qž øü ŸJ-e•^MPc
MP_XbOPMm0xc1N^J<)1
,e¼“,QžÌ}Ï ú ¾ÅÏ » ÐlÑkÍ ü Ÿ¸ý,e¼“,Qž ̢Рú ¾ŒÏVÐ » ÑkÍ ü Ÿ¸þ,e¼“,QžÌ}Ñ ú ¾ŒÏVÐlÑ » Í ü ŸJÊ
Ä 9Zc0QLNC.01R^J<u1
,e¼“,Qž Ì ÏDú¾ŒÏ » ÐlÑ Í ü Ÿ ·ÿ ,e¼“,Qž.Ï » Ì Ï8¾]Ð2Ñ Í ü Ÿ
·ÿ ,e¼“,Qž.Ï » Ì ,e¼®Ðv¼ÂѾ]ÐlÑ Í ü Ÿ
·ÿ ,e¼“,Qž.Ï » ¡ ,e¼®Ð2¤2¡,¼ÂÑQ¤ ü Ÿ
·ÿ ,x, ž ü Ï » ¡,e¼Ð2¤k¡ ,e¼Ñk¤lÊ
VMm_XMPOP<BLNOPnh¨r0`_X<„n?@.9V1R<BMmA
, ü Ð » ¡,e¼ÂÑQ¤2¡ ,¼ÂÏV¤l¸ , ü Ñ » ¡ ,e¼ÏV¤2¡ ,e¼®Ð2¤2Ê
,xž ,xž
´HOÈ1RMmbJOmnDMPAoX1N^0xc081R^LR0k08MmAJ0QFIHJ<)OPMm1NMP0QcQW¨v08o.021
Ï » Ð » Ñ » ¡ ,e¼ÏV¤ » ¡ ,e¼®Ð2¤ » ¡ ,¼Ñk¤ » ,xž, ú Ê
¥ @u¨v0QCB0QLQWVŸ ü Ï ü ,8MP_XbOmMP0Qce1N^J<)1Ï;¡,e¼ÂÏV¤rë”,„‰ Ú - ¥ 0kAZ>20
Ï » Ð » Ñ » ¡ ,e¼ÏD¤ » ¡,e¼Ðk¤ » ¡,e¼ÂÑQ¤ » Ì Ïá,e¼ÂÏV¤ Í » Ì Ðu¡,e¼Ð2¤ Í » Ì ÑB¡ ,¼Ñk¤ Í » ë ,Qž, ú ¸
<X>k@BAI1NLd<BKVMa>l1RMm@.A·- ƒ 0=>k@BAJ>kOmHZKV01N^J<)1e1N^0`o.MmC.0kA½0QFIHJ<u1RMm@.AŠ^Z<BcLR0Q<)OÔLR@D@)1dck-

Ú
   "!#$&%' ()! +*-,. ,0/12(4356+75982$&82":!)%')'$<;=5>$&7?
A@4B@<C$4";8"4% D" E 7 (,: F,GBH943)I8$48")!:%')'$&;J56$47?1/K7L+B,:+A,MI*-,G(.
?N43"?O%'QP=@<R@&ST@4UV$4";482"4%'D" EXWH$&7YDF43:NP=@<R(@&SZ@4U[$4]\<^_>_a`6b2c4def

gihkjmlnpo6hkqXrs :;$4D+43:II@<R(@4/t@4P$4 7:EM'%uXvN":

w FR PZx w +/FPCxzy{|~} w /+PCx w B€

 %u? %''$&'E43)F7)EG'%'% ‚E]75ƒ/=@&SZ@<R(@4„% D";

w R U†xzy{|ˆ‡ w /AR S‰xzy{|X‡ w / SŠxzy{|ˆ‡‹y{|~} w ŒxK"{|=} w ]€

Žt3M);~9 7)4%' w FR P‡ w +R+U‘xK"{ | H’“=5>7''7YN;=43:IR(@4U-@4P”'%'I7•43F;4?A'%'

A Q

F M

B P D C

– #  Z;4%'? %'u$& E—8)$47pD˜43) w PARŒ‡ w (PFS xŠ"{ | ™Žt3)%';š%'? 8:'% ";]43)•P=@&SZ@<R›$&


7'u%'"$"ƒŽt3M);tSZ@4Uœ743•u%  7Q43F'%uH7%')%uPž)!-R

ŸMJ%')!•43L_'c4d¢¡£¤87;4;4%':  ¢D u 75H¥ ‡¤¦p,2N3$4F¥k@&¦+$4A87;4% 4% DF%u§$&;I;4)<3¨43:A!)% DG%'!";


¥+‡©"ª¦ )!˜"ª !)% DG%'!";~¥ z ‡ "¦Y

g hkjmlnpo6hkqXr  %')]"ª]!)% DG%'!";¥F‡z"¦p,i9;43)("ªB!)% DG%'!";t¥}—Ÿ¦ )!«3")(% I';7š!)% DG%'!";


i
¬¥(}"Ÿ¦pNŽN3)%';t%'¨4$&-%'?8:'%'";~43)A"ª)­¯®°¬¥A‡¦±<  %'? %u'$& E¨§­¯®m¥‡©"ª¦±9x²³§­¯®m¥F‡Ÿ¦±9x²
§­¯®°¬¥(‡T"Ÿ¦±tx´²Š§­¯®°¬¥‡¤¦±<  %u)F!µ®¶@""ª±x·,i97)')!A43)¸§ª)­¯®°¬¥(‡¹¦±<+Žt3º:;
9 ? "Ešt$<% F¬¥ ‡‹¦=xK¸§ª»Q567$N;7?A)4$&iM)?A2$N»2ˆvN")

¬¥ ‡‹¬¦txK¸§ª»F‡¤¼¦txK¸¸§»A‡‹¬¦9}®°»A‡‹¦±<€

Žt3:%';~;437Y;=43)¬!:% DM%u!";=» ‡‹¦ )!Q3")» ‡‹¦ ½¬M –  43)$45>7$& 7)4%'

¬®m¥ ‡‹¦±ƒxK¸§ª»A‡‹¼¦txŒ"ª»F‡‹¼®°»F‡‹¦±~½Z"ªN‡‹¼¾¨¬AxK"¬M€

’“A567' 7pN;A43)¥(‡z¦¨½™ŸM˜Ž0¿G%'«¥ÀxÁŸª-:!—¦BxÁ¼M,ƒˆQ;B43)(43š7)!)% 4% 7:;F75t43)


8)$&7: "?1$4F;44%u;Â)"!iHŽt3M);~43A? %')%'?)?D¢u+75ƒ¥ ‡‹¦I%';ŸM

ªMN’“5µ¥k@&¦p@4Ã+$4 43$&+827;&% 4% D+$4"´M)?A$&;,G8)$47pD 43)

¥ºÄH‡© ¦<ÄH‡© ÃÄX‡©


‡ ‡ ½ªM€
¦ƒ‡—à ÃX‡—¥ ¥ ‡‹¦

gihkjmlnpo6hkqXr – ();F43)F$&% DG%'´%u"ź:'% 4% ";=¥MÄX‡z½zŸ¥:,¦<Ä9‡z½ÆŸ¦+:!-ÃĈ‡©½zŸâtvN")


9+7)4%'
¥ Ä ‡© ¦ Ä ‡© Ã Ä ‡Æ Ÿ¥ Ÿ¦ ŸÃ
‡ ‡ ½ ‡ ‡ €
¦ƒ‡—à ÃH‡—¥ ¥+‡‹¦ ¦ƒ‡—à ÃH‡—¥ ¥ ‡¤¦
Ÿ ¥
 Ÿ ¦
 Ÿ Ã

‡ ‡ ½ªM€
¦ƒ‡—à ÃX‡‹¥ ¥+‡‹¦
Ç !)!)%')(¬(743•;&%'!";,G43)%';=%u;="ź:% D¢'"§=7

®°Ÿ¥ ‡‹Ÿ¦ƒ‡‹ŸÃ"±~È ‡ ‡ ½yM€


¦H‡Éà Nj¥ ¥ ‡‹¦ºÊ

Žµ¿M%u˚xz¦ƒ‡—â,)Ì x©ÃH‡—¥k,:ÍAx©¥ ‡‹¦p,43)%u;=%';="ÅM)% D "§=7

®6Ë(‡ÉÌ ‡—ͺ±~È ‡ ‡ ½yM€


Ë Ì Í Ê

Žt3:%';~%';t7);"ÅM")F75 Ç Î “Ï Î %'"ÅM)'% ‚E

Кj>nYÑMÒ¢q0ÓnYÑGj>ԃr Žt3) ;4:;4%'44% 7);=¦ƒ‡—ÃNxÆË0,)ÃX‡—¥(x©Ìk,:¥+‡‹¦=x©Í '"!);=7

Õ Ÿ¥ Õ Ì ‡ÉÍF}Ë Õ Ë Ì
x x È ‡ }#ª½¹¬+}#ª xzªM€
¦H‡—Ã Ë Ì Ë9Ê

¸ Ç ¬F¾]¬;4ÅM)$4 %';9!)%';4;""!š%']7LyF$4"4 ";=MEL'%u";X8k$&' ":7(%'4;9;&%'!";9;4)&3B43)='243";4


$4"4) ";N3)YDF%'º$;4%u!";ƒWH$47pDF43)N43$4F$4 '9YEM; n<Ö(h 7$<"§N$4"4 ";

gihkjmlnpo6hkqXrt× 7);4%'!$~43 !:%';4;"4% 7B75µ43+% D"•¬¾•¬(;4ÅM)$4+%'B7L77$&"º=$4"4 ";


N%'43«'"; 827;&;4% : $4";Žt3(7)'E¨$4"4'N% 43Ø$4"%'; ¤¾—($4"4   %'? %u'$& E,
9  ¾šŸM,´¾šªA$4"4 ";t5>7$=$4";NŸ],Ùª():% 4;J’“B43+;+750¸))% 4;99 ?L"Eš3)"D+"% 43)$
BN¾¸($4"4)  7$~Ÿ+¾BŸ;4ź:$4  %'? %'u$& E,43$&Iš ]¾L¼A$4"4  567$~$&"(¼):% 4;
)!¹¾#¬Q7$ Ÿš¾˜ª¨$4"4 B567$(¬¨))% 4; Ç ºE$4"4 BN% 43À$4"ØÚ¨))%'4;F?);F¨¾ÉÚ
$4"4) ,iN3)%'<3-%u;7 87;4;4%': F;4%u)A43)(u$4"; ;4%'!(7)'!-(¬B))%'4; Ç )!غE•$4"4 
N%'43$4"F+):% 4;ƒ?A:;ƒtFŸ~¾+¸ $4"4) t’“5k43$&N%';ƒºE(!:%';4;"4% 7(7543N% D"L¬~¾F¬F;4ź:$4
%'¨7šy]7G7)$&"º$4"4) ";IN% 43¨$&";+¥)Û+Üz¥ Üz¥MÝAÜz¥ºÞÜz¥MßÜz¥ºàÜz¥MáAÜz¥MâAÜz¥Mã¢,
Ä
43"•ˆ 7k;$4D+43)

¥ Û ½T@0¥ ½ŸM@0¥ Ý ½ªM@¥ Þ ½‹¸@¥ ß ½‹¸@¥ à ½¼M@¥ á ½¤¬M@0¥ â ½¬M@¥ ã ½M@


Ä
)!Q3)") 43+74µ$4"75J'i43+$&"4 ";N%u;

¥ Û ‡—¥ ‡Æää䢇—¥ ã ½TX‡‹ŸN‡¤ªN‡À¸I‡É¸I‡¤¼t‡‹¬N‡‹¬t‡‹Axzªy吪¬M@


Ä
N3:%'&3š%';943+$4"75043 % D"•;4ÅM)$4Hv") % 50 ¬F¾Q¬(;4ÅM)$4+%';~!)%';&;""!Q%'B7Ly$4"4 ";
;t;44% 8:)'"!Q%uš43 8)$47k "?Q,G43$4 ?);=+97]7)$&"ºt$4"4 ";"

¼MI +*[]«Åº:!$&%''$<X%'#N3)%'<3ÀIæ%';A8:$&''"H7#+*ç)!À8$48")!)%':'$+7«I*-è
Ižxzª+*-è)!B43+$4"7543+ÅM)!$&%'u$&k%u;ˆ¸ƒ’“50(% $< Iš !$&YNB7)&3)%uuk43)
;4%u!";~75043)Fź:!$&%''$<°,GÂ:)!š% 4;=$&!:%');

gihkjmlnpo6hkqXr 9P,)R(,é(,)ê4387%'º4;H7507§4=75´%'% $& t% 43 43 ;4%'!)";ˆIB,M (,+*-,


*B„$4";8"4% D" E  %':F*†%';N8$48")!)%')u$ˆ7BO:!•ë%';=8:$&u "i7]*Q(,:ˆ(;F43:
IPÁxO êCxëêJ*›x™*Béëx1ìG,µ43B$&!)%'); 75=43]%':;4$&% "!Ø% $& •A(PÁxO Rëx1̘:!
ARTxT+éZx©ËHíI;4%' Ižxzª+*-,ˆFtìN‡ÉÌxTª®6ì‡À˱<

Ÿ
D r R x C
x
Q
r

S y

A r P K y-x B

 %u) 43F$4"75JI *î%u;~¸,)ˆA';7

¸Ax I*«®mIT‡¤+*Q±ƒx ®°Ÿ¢ì§±ï°¸:®6ì‡ÉËi±¶ð€


Ÿ Ÿ
Žt3M);Nˆ7)4%'«ì)®6쇗Ëi±=x·IíI;4%'BWHEM43)7$&; 437$&"?Q,ˆ7)4%'« Ä x„ñ Ä ‡¹+ñ Ä 
v7Y9D$F ŒxCÌA‡‹Ë0,2(ñçxCÌ }#˘:!Ø ñ[xKŸ¢ìG  :;4%'44%'43";4(:!-;4%u?8:'% 56EG%',9
HËkÌx©ì Ä ƒò~Hì‡BÌxzª®6ì‡B˱µ%'D";HÌ xzŸ¢ì‡Qª¢ËƒŽt3M);Jì Ä xÆ˵®°Ÿ¢ì‡Qª¢Ëi±ƒ)!(43)%u;H;4%'?8k'% Â)";
7#®6ì}—ª¢Ëi±®6ì+‡¤Ë±Ix”{G – ]7)')!(43)+ìšxª¢Ë0óN7pž43$4"'4%'7Øì®6ì ‡¤Ëi±xë%'?8k'% ";
43)N¸ìÄtxzªM,:% DM%u(ì xZô ªõŸM

¬MWX$47YDI43)~43$4 $4 %'Â:)%'" EA? ºE 87;4% 4%'D%'º$&;9ö-;4:&3]43)~öX®6öA‡—Y±ƒBN÷G8)$4";4;4"!


;9A;&)?”75i‚ˆ7(87;4% 4%'D;4ÅM)$4";9%'-d£ƒ_ c&d¢¡f£k97!)%¯ø$4"ºX~"EG;®mv$4I¥MÄ´‡Ø¦fÄN)!š¦fÄi‡«¥MÄN$&
7);&%'!$4"!Q;=43)F;4? $&8)$4";"º44% 7iù±

g hkjmlnpo6hkqXr R1xžöX®6öQ‡CY±<’‚ %';+7ºD")% "º  7•<37º7;4 ö—x”ú Ä ,´567$+43"ÀR™%u;I'$4"!EØ


i
;4:?ë75ˆ97•;4ź:$4";û R”xKú Ä ïüú Ä ‡C"ðx‘ïüú Ä ð Ä ¤ ‡ ú Ä  ’“5H56)$443$ ú Ä %'4;" 5H%'; š;4)?Á75X‚ˆ7
;4ÅM)$4";",);4"Ešú•ÄtxÀýkÄX‡—þÄ¢,43"

RCx™ï¯ý Ä ‡‹þ Ä ð:ï°ú Ä ‡©"ð x™ïaý:ú‡—þð Ä ‡žï¯ý]}˜þ¢ú¨ð Ä €

ó7Ø43)]43)-‚ˆ7‹$48:$4";"º44% 7);B5>7$•RV$4Ø!)%';4%u)CŽt3M);,=567$š÷G? 8: ,t9«?L"E4¿


úîxz¼»2,§ýšxzª»2,2þ+x©¸§»2,:N3$&F»šD¢$<% ";=7YD$ )4$<iº)?A$&;ƒ’¶Q43)%';=;4Fö-xÆú Ä xZŸ¼» Ä ,
)!
RZxO» Ä ð Ä ‡žïü¼»ð Ä x™ï&"¼» Ä ‡É¸§»ð Ä ‡KïŸ{» Ä }#ª»ð Ä €
Ç ; ˆ(D$4E»¨7pD$A)4)$&Jº)?A$&;,9 %'Â:)%'" EQ? §E«º:?F$&;I75943(5>$47?†öX®6öQ‡CY±
"<3•75JN3)%'<3QQ+÷M8:$4";4;"!•;;4)?™75µ97];&ź)$&";t%'š97]!:%';4%')~~"EG;

Úºtÿœ+43A;t75ƒ'i87;4% 4%'D %'§$<;=$4"$I43)•7$"ź)´7] )!¨ «û§ÿ[ÿV 


5m))4% 7Q;&)&3Q43:
 ƒ®6ˇÉÌ)±Hx ƒ®6Ëḵˆ567$N'Ë«½‹¸,)ÌQ½‹¸H’“5ƒ®°±ƒxTyM,:!$&? %uƒ®°y±<

gihkjmlnpo6hkqXr –  7:;$4DF43)

ƒ®°y±ƒxƒ®6¸I‡—¼±ˆxƒ®6¸+äY¼±Hxƒ®°Ÿ{§±Hxƒ®¶"¬N‡É¸º±Hxƒ®¶"¬+为±Hxƒ®°¬¢¸º±
xƒ®° äY±Hxƒ®°N‡—±Hxƒ®¶"¬±ˆxƒ®6¸+为±Hxƒ®6¸I‡—¸º±Hxƒ®°±<€

v ")% 5 ƒ®°±NxžyM,´43" ƒ®°y±xžyMš®mŽt3)%'; %';I7;$&%uFŽt3$&? YE-(743$ !)% ø2$4"ºI9YEM;



75N8:8)$47&3:%' ƒ®°±F56$47? ƒ®°y±<˜Žt3]%u?87$44§A43)%'«7«2B7:;$&D"!É%u;F43B5m43:43)
$&:  ƒ®6Ë(‡—Ì)±H x ƒ®6Ë:Ì)±98)8:'% ";~7: EšN3"šËØ)!•Ìš$4FI ";t¸ s  ? YEQ;$<%';t);&%'
M)?A2$<;~Ë-)!QÌBN3)%u&3Q$4F;&? ' $~43)¨¸,):~43)N%u;t7=D'%'!iƒ7$÷G?8k 

ƒ®°y±Hxƒ®°ªIäpª±Hxƒ®°ªt‡‹ª±Xxƒ®°¬±Hxƒ®6¸‡‹Ÿ±Xxƒ®6¸+äpŸ±Hxƒ®°±<@

%';=)7N(D¢u%'!Q;$&%')ù±

ª
    !"# $&%

')(+*,.-0/&12435,7698:6); <=->,.?@6)8BADCE, F-HGJIK6)8=ADCL,9MN/PO35,&->QB,73BILRJ, ;.-HSDGTS9U VW1X/P2ZYWIE->Q:OSD8[12M


698BF[1\]35,7-HQ=,^69CE-HI_-H<BF=,PU`G>SDab1cS)8[/P2(edNQ=SfYg-HQh6i-&VW2j\XOlknmpo)qr(
sutwvyxzf{`tw|~}
 G>6Y  *n€5,rG>€w, 8BF=IL; <=CK6)G7-HS/&1M"O‚ €5,rGƒ?
€w, 8=FBIL;r<BCK6)G-HS„/P2Mj6)8BF O† €w, GJ€5,r8BFBIL;r<N?
CL6)G-HS‡1\ˆ( ‰0QB,r8Š\† ‹ O‚Œ( dNIE8B; , A

/POc3=ILR>,r;r->RTV0/XpYŽ,SD3BRJ, GƒD,P->Qh69-ŽV1^/&O‘‹
VW‚’/&O“( ‰0Qp<BRZIL8”-HGJIK6)8=ADCL,rR‡/&•"O 6)8BF
/P‚O“Ye,–RJ, ,‡->Qh69-—VW•˜/&O ‹ V‚’/POM
E

V0/&‚O™‹›šDœDqP‹]VW/P•~O“M698BFž/&O”ILRP; SDaŸ?
aSD8 ( ¡, 8B;r,—->G>IL6)8BADCE, R¢/P•"O 6)8BFŠ/&‚O
K
M

69G>,Œ;rSD8BADGJ<B, 8£-XADIE¤IL8=A¥O• ‹ O‚Œ(§¦Ž<=- L

O• k O†4¨R>IL8=; ,41\ CLIE, R¥IL8=R>ILF=,–-HQB,


->G>IL6)8BADCE,£©ª3£«”6); <=->, 8B,rR>R4€BG>SD€w, Gƒ-@«=¬.( ‰0Qp<BR
\X† k‡O†4(W‰0QBILR+ILa€BCLIE, RŽ->Qh69-­VW\XO† k
B D C

VWO\X† ‹ šDœ q7® VW\XO†4(4¯¢,;rSD8B; CE<BFB,


->Qh69-°VW\XO† k mpo9q ( dNIE8B; , VW2j\XO ‹
VW\XO†4B-HQB,­GJ, R><=CE-0U`SDCECLS±YWR (
²v³z±´pµi|·¶Nz±´žsutwvyxzf{`tw|~}
*,r-V2j\XO¸‹c¹N(¯¢,Qh6D,2^O§‹cº¤».¼B©ª»Ž½—¾r¬
6)8BF2^\¿‹gºe; S)RÀ2(˜ÁR>IE8BA^RJIL8B,WGJ<BCL,WIL8-HGJIK6)8=ADCL,
A 2j\XOBYe,­Qh6D,
2jO ‹ 2^\
RJIL8¹ RJIL8©ª2¥½Ã¹)¬¤Ä
E

c θ b

‰0QBIERGJ, FB<=; , RW-HS


©y»¤½¾r¬=R>IL8¹ ;rSDRÀ2¿‹‡»5RJIL8W2¢;rSDRÀ¹ ½»w; SDRw2¢RJIL8W¹ Ä
d¤ILa€BCLI_Åh;69->ILSD8ADI_D, R¾uRJIL8W¹ ;rSDRÀ2¿‹‡»5RJIL8W2¢;rSDRÀ¹F[R>S-HQh69-
B D ab/(b+c) C
a

-H6)8W¹j‹ ¾u»5;RJILS)8Rh22 ‹ ;RJILS)8WRÀ1 2 ‹ R>IE8©`ÆÂR>IL¼D8Ç 1 ® 2X¬ Ä


d¤IL8B;r,0/&12–IER"6); <=->,.?@6)8BADCE, F DYe,WQh6D,/ZÈÃÆ·¼DÇN(¡+, 8=; ,+1ž½“2›kÃƼDÇ&SDG"1ckÉ©`Æ·¼DÇ)¬ ® 2(
0‰ Q=, G>,.U`SDG>,­RJIL81ckÃR>IE8©`Æ·¼)Ç ® 2X¬Ê(˜‰0QBILRWILa€BCEIL, R~-HQh6i--H6)8W¹Ÿk‡'^6)8BF:QB,r8B; ,^¹ŸkËÆ·¼9mB(
Ǥ(+*,.-jºÀÌH»fÌH¾X35,Ÿ->QBG>,r,8B69-H<BG>6)C 8p<Ba3w, GJR­R><B;ÍQž-HQh6i-jº¨ÈZ»È!¾698BFžAD; F·©ª¾ ® ºÀÌH¾ ® »r¬P‹l')(
d¤<B€B€wSDR>,-HQ=, G>,:,ÊÎ=IERJ->Rˆ698°IE8£-H, A), GϞR><B;ÍQ°-HQh69-º½gÏÀÌH»W½‡ÏhÌ;W½–ÏžU`S)G>aÐ->QB,:R>IEFB, RŸS)UP6
GJILADQ£-ƒ?Ñ6)8BA)CL, Fˆ-HG>IL6)8BADCE,)(˜Ò˜GJSf),j->Qh69-0->QB, GJ,^,ÊÎ=IERJ-WIE8p->, AD,rG>RÓ@ÌHԙRJ<B;ÍQ“->Qh69-+¾˜½°Ï‹‡Ó³Õ½°ÔÕ±(
sutwvyxzf{`tw|~}
¯Ö,­Qh6D,
©y¾ ½ÃϤ¬ Õ ‹!©ªº7½ÃϤ¬ Õ ½g©y» ½°Ï¤¬ Õ Ä
‰0Q=ILR0G>,rFB<B;r, R+->S
Ï Õ ½ËÇ)Ïw©ªº&½Ã» ® ¾ ¬½°º Õ ½°» Õ ® ¾ Õ ‹gœ Ä
¤d SDCE¤IL8=A-HQB,­×p<h6)FBG>69-HIE;7, ×p<h69->ILSD8U`SDGWÏhBYe,­SD3=-Í69IL8
ÏX‹ ® ©yº0½¢» ® ¾ ¬ÀØ Ù ©yºP½Ã» ® ¾ ¬ Õ ® ©ªº Õ ½°» Õ ® ¾ Õ ¬"‹ ® ©ªº0½¢» ® ¾r¬ÀØ Ù ÇB©y¾ ® º¤¬ ©y¾ ® »r¬ Ä
d¤IL8B;r,+ÏjILR"6)8IL8£->, AD,rG ¤ÇB©ª¾ ® º¤¬ ©ª¾ ® ».¬~a<=RJ-~35,67€w, GJU`,r;r-~R>×p<h69G>,)MNRH6«ˆÇB©ª¾ ® º¤¬ ©ª¾ ® ».¬e‹—ÚÀÕ±
¦e<=-WAD; F·©ª¾ ® ºhÌ; ® ».¬"‹4'D("¡, 8B;r,jYe,­Qh6D,
¾ ® ºŸ‹–Ç9Û Õ Ì ¾ ® »W‹—Ü Õ SDG ¾ ® ºŸ‹ Û Õ Ì ¾ ® »0‹‡Ç9Ü Õ Ì
YWQ=, G>,0ۓknœ76)8=FÜknœ7698BFAD;rF·©ªÛÂÌHÜN¬ ‹4'D(Ý@8Ÿ, I_-HQB,rGS)Uh->QB,T;69R>, R"Ï‹ ® ©yº½» ® ¾ ¬fØÇ9ÛÀÜÀ(
Ý@8:-HQ=,­ÅBG>RJ-; 6)R>,
¾"½ÃÏ‹gÇ)¾ ® º ® »˜ØnÇ9ÛÀܟ‹gÇ9Û Õ ½°Ü Õ ØËÇ9ÛÀܟ‹›©`ÛØËÜN¬ Õ ½ÞÛ Õ Ä
¯Ö,7SD3BRJ, GƒD,­-HQh69-WÛ‹ ܁IEaˆ€=CLIL,rRe->Qh69-WÛ‹ ܟ‹4'j6)8BF:QB,r8B; ,^¾ ® ºŸ‹–ÇNÌ; ® »0‹¿'D("¡+,r8B; ,
ºhÌÍ»±Ì;X6)G>,Ÿ-HQ=G>, ,; S)8BR>,r; <=->IED,IL8£->, AD,rG>R (Ÿ¯Ö,6)CLRJS[RJ, ,-HQh6i-j¾T½—Ï‹l'ŸU`SDG>;rIL8BA[»e½ ϓ‹cœN
;rSD8£-HG>6)FBIL;.-HIE8BAj->Qh69-Ž»w½žÏIERŽ6jR>IEFB,+S9UÂ6j->G>IK698BADCL,9(‰0Qp<BRŽÛŒ‹g ß Ü6)8=FQB,r8B; ,P¾u½ϟILR˜-HQ=,PR><=a
S9U-@YeSˆ8=SD8N?@àr, GJSˆIE8p->, AD,rG0R>×p<h6)GJ, Rr(
d¤ILaILCK69G>CE«)NIE8“->QB,XRJ, ; S)8BF’;69R>,^Ye,AD,.-&¾"½nÏ‹–Üp՘½–©`ÛØnÜN¬ƒÕ(0‰0Qp<BR7¾"½nψILRW-HQB,^R><BaS)U
-@YeSR>×p<h6)GJ, R (
²v³z±´pµi|·¶Nz±´žsutwvyxzf{`tw|~}
á 8B,Waˆ6«<BR>,;ÍQh6)GJ, ;r-H6)G>IERH69->ILSD8S)Uu€BG>IEaˆI_-HIE),0Ò «p-HQh6)A)SDG>, 6)8ˆ->G>IL€=CL, Rr( á 3BRJ, GJ),+->Qh69-~AD; F·©ª¾ ®
ºhÌ; ® ».¬&‹l'ILa€BCLIE, R-HQh69-j¾Ž½¥ÏÀÌHº^½¥ÏhÌÍ»e½¥Ï6)GJ,ŸG>, CL69-HI_D,rCE«’€=G>ILa,)(¡+, 8=; ,Ÿ-HQB,rG>,,.ÎNIERJ-
IE8£-H, A), G>RWÔâkËãžR><B;ÍQ[-HQh6i-
º&½°ÏŸ‹ Ô Õ ® ã Õ Ì » ½°ÏŸ‹‡ÇiÔãÌ ¾˜½ÃÏX‹gÔ Õ ½Þã Õ Ä
äN(+åIL8BF6)CECh€h6)ILGJR+©yºhÌÍ».¬ S)UG>, 6)Ch8p<Ba3w, GJRŽR><=;ÍQ-HQh6i-~YWQ=, 8B,.D, GTæžIER~6^G>S¤S)-"S)U5Ú Õ ½’ºDڏ½’»0‹‡œN
æ·Õ ® ǟIERW6)CLRJS6GJSNS9-WS)U->QB,­, ×p<h69->ILSD8u(
sutwvyxzf{`tw|~}
ç S)8BR>IEFB, GW-HQB,j, ×p<h6i-HILS)8“ÚÀ՘½Ãº£Ú½Ë»‹¿œN(0Ýè-PQh69R-@YeSG>S¤S)->R±©ª8=S)-8B, ;r, R>R>6)G>IECE«[G>, 6)C³¬ÊhRH6«“æ
698BF:é~("ê˜I_-HQB,rG+枋 é¢SDGW楋g ß é"(
ë^¶hìr´„íh}
d¤<B€B€wSDR>,:æZ‹âé""R>S-HQB69-æ–ILR6’FBSD<B3BCE,G>S¤S)-(¢dNIE8B; ,:æ·Õ ® ÇILR69CLR>S’6žGJSNS9- -HQ=,SD8BCE«
€wSDRJR>IL3=ILCLI_-@«XIER"樋gæÕ ® ÇN( ‰0QBIEReGJ, FB<=; , R~-HSˆ©yæj½Ö'±¬ ©yæ ® ÇD¬"‹‡œN( ¡+,r8B; ,P枋 ® '+SDG~枋–Ǥ(
á 3BRJ, GƒD,­-HQh69-+ºX‹ ® Ç9æ„6)8BF“»0‹‡æÕ (˜‰0Q¤<=R©ªºÀÌH»r¬"‹›©yÇNÌ'f¬ŽS)Gj© ® mBÌHmp¬Ê(
ë^¶hìr´žî }
d¤<B€B€wSDR>,¨æ ‹ ß é"(Š‰0QB, GJ,Ö69G>,U`SD<BG€wSDR>RJIL3BIECLI_-HIL,rR M©`݃¬æ⋠æÕ ® ÇÞ698BFgé‹ éÂÕ ® ǤM
©`Ý>݃¬æ⋠é·Õ ® ÇÞ6)8BF‡él‹ æ·Õ ® ǤM©ªÝJÝ>݃¬æ⋠æÕ ® LJ‹ éÂÕ ® ÇÞ6)8BFgæ ‹ ß é"M^SDG
©`Ýèï­¬Ñ鋇æ Õ~® Ç­‹gé Õ"® Ç6)8BF[旋 ß é
©`݃¬T¡+,rG>,©yæeÌ>鬋›©ðÇNÌ ® 'f¬ŽS)Gj© ® 'DÌÊÇD¬Ê("¡+, 8=; ,©yºhÌÍ».¬"‹bñ ® ©ªæ:½°é¬.ÌHæé·ò‹›© ® 'DÌ ® ÇD¬.(
©`Ý>݃¬0dN<=€B€5S)R>,^枋géÂÕ ® Ç6)8=F[é‹gæÕ ® ÇN(˜‰0Q=, 8
æ ® é‹gé Õ ® æ Õ ‹›©`é ® æ¬ ©`é½°æ ¬ Ä
d¤IL8B;r,­æ ‹— ß é"=YŽ,jAD,.-+遽Ã樋 ® 'D(~¡SfYe,r), GrwYe,X69CLR>SŸQB6±),
æ:½°é’‹gé Õ ½°æ Õ ® m^‹›©yæ:½Þ靬 Õ ® Ç)æ·é ® m Ä
Ç
‰0Qp<BR ® 'Ÿ‹b' ® Ç)æ·é ® mB YWQ=IL;ÍQžILa€BCLIE, RW-HQh6i-­æé°‹ ® 'D(j‰0QB, GJ,rU`SDGJ,©ªºhÌÍ».¬P‹ ñ® ©y撽
靬ÊÌÍæé ò ‹!©ƒ')Ì ® '±¬.(
©`Ý>ÝJ݃¬­ÝèUæ‡‹æ Õ ® ǒ‹é Õ ® Ç698BF„æb‹ ß é"->QB, 8Þ懋 ® é"(‰0Qp<BR斋ÇN ég‹ ® Ç[SDG
枋 ® 'D=鞋Z'D(˜Ý@8:-HQ=ILRW;6)RJ,ˆ©yºhÌÍ».¬"‹›©yœ=Ì ® mp¬T698BFÖ©yœ=Ì ® 'f¬Ê(
©`Ýèï­¬PóS)-H,^-HQh69-&鰋›æÕ ® Lj‹!éÂÕ ® Ç6)8BFžæZ‹! ß é°ILRPIEFB, 8£->IL;69C-HS¨©`Ý>ÝJ݃¬.uRJS-HQh6i-&YŽ,AD,.-
,ÊÎ=6);r->CE«R>6)aˆ,7€B6)ILGJRj©yºhÌÍ».¬.(
‰0Qp<BRWYe,jA),r-+ôŸ€h6)ILGJR M©ªºhÌÍ».¬~‹!© ® mBÌHmp¬ÊÂ©ðÇNÌ 'f¬.Â© ® ')Ì ® Ç)¬.·©ƒ')Ì ® '±¬.Â©ªœ=Ì ® mp¬.Â©ªœ=Ì ® 'f¬.(
m=(+¡S±Y–aˆ6)8£«ôi?èFBILADI_-T8p<Ba3w, GJRP69G>,&-HQ=, G>,jR><=;ÍQ“-HQB69-õ
©ª6£¬-HQB,­FBIEADIE->RTS)U,6);ÍQ“8¤<=a35,rG6)G>,j6)CEC5U`G>S)a‘-HQ=,jRJ,r-jöp'DÌÊÇNÌHä=ÌHmBÌÊop÷pM
©`3À¬:6)8£«:FBILADI_-Ž->Qh69-+6)€B€w,6)GJR0IL8-HQ=,j8p<BaX35,rG+69€B€5, 6)G>R69-WCE,6)Rƒ-W-@YWIL; ,ø
©ªêÎB69aˆ€BCE,)õ~ÇDÇ)oDÇDoDÇXIERW6)8[6)FBaILR>RJIL3BCE,&8p<Ba3w, GrBYWQBILCE,jÇDÇDÇN'±ä)äILR08BS)- (ù¬
sutwvyxzf{`tw|~}
d¤IL8B;r,7,6);ÍQ“FBILADI_-TS¤; ;r<BG>R+69-WCL, 6)RJ-0-@YWIE; ,9BYŽ,jQh6D,­U`SDCLCES±YWIL8BA€wSDRJR>IL3=ILCLI_-HIE, R õ
íhú ‰0QBG>,r,ˆFBIEADIE->RPS¤; ;r<BG7-@YWIL; ,Ÿ, 6);ÍQ (Ÿ¯Ö,Ÿa6«’;ÍQ=SNSDRJ,Ÿ-HQBGJ, ,FBILA)IE-HR+U`G>SDa§öp'DÌÊÇNÌÍäNÌHmBÌÊo¤÷IL8
û o ‹›'±œYT6«¤R (ÝèU˜,69;ÍQ’SN;r; <BGJRP,ÊÎB69;r-HC_«-@YWIL;r,)5->QB,j8¤<=a35,rGPS)U R><B;ÍQ69FBaˆIER>RJIL3BCE,­ôi?@FBIEADIE-
äDü
8p<BaX35,rG>RWILR ô=ý
ÇNý¤ÇNýpÇNý þ '±œj‹‡šDœDœ Ä
î ú ‰TYeSFBIEADIE->RjS¤; ;r<BG­-HQBGJ, ,ˆ->ILa, Rj,6);ÍQu(:¯Ö,; 6)8Œ;ÍQBS¤SDRJ,Ç[F=ILADI_-HR­IL8 û oÇ ü ‹™'±œ[YŽ6«¤R (
¡, 8B;r,j->QB,­8p<Ba3w, GWS)U6)FBaILR>RJIL3BCE,&ôi?èFBILA)IE-T8p<BaX35,rG>RILR
ô=ý '±œj‹–Ç)œ)œ
ä=ýpäNý˜þ Ä
ÿ ú á 8=,F=ILADI_- S¤; ; <=G>R"U`SD<BG -HIEaˆ,rR"6)8BFŸ-HQ=,+S)->QB, G -@YWIE; ,9( ¯¢,6)GJ,0;ÍQBSNS)R>IL8=Aj-@YeS^F=ILADI_-HR˜6)AD6)IL8 
YWQ=IL;ÍQ:;6)8[3w,7FBSD8B,­IL8ž'±œXYŽ6«¤R ( ‰0QB,^-@YeSˆF=ILADI_-HR06)GJ,7IL8£-H,rG>;ÍQh698BAD,693BCL,9(e¡, 8B;r,7-HQB,­FB,rR>ILGJ, F
8p<BaX35,rGS9U 6)F=aˆIER>R>IE3BCL,Pôi?@F=ILADI_-T8p<Ba3w, GJR+IER
Ç þ mBô=ý¤ýǤý˜þ '±œ^‹gäDœ)œ Ä
 ú åIL8h6)CECE«ˆ6)CEC FBILA)IE-HRT6)G>,&->QB,­RH6)a,)("‰0QB,rG>,j6)G>,^oR><=;ÍQ8p<Ba3w, GJR (
‰0Qp<BRW-HQ=,­->S)-Í69C 8p<Ba3w, GWS)U6)FBaILR>RJIL3BCE,P8p<Ba3w, GJR+IERWšDœDœW½ÃÇ)œDœ½ÃäDœ)œW½Ëoj‹Z'm£œDoN(
o¤( ‘-HGH69€5,rà IL<=a /&1X2^O˜IE8ŒYWQBIL;ÍQÞ/P1¸IER€h6)G>6)CLCE, C"-HSž2jO"ILRIE8BR>;rG>IE35,rF„IL8Þ6’; IEG>;rCL,YWI_-HQ
;r, 8£-HGJ, ˆ(+dN<=€B€5S)R>,X->QB,XFBIL6)ADSD8B6)CLRW/&2Š6)8=F1O™S)U-HQB,j-HGH69€5,rà IL<=a IE8£-H, GJR>,r;r-76i-­†4u6)8BF
X† ‹–ÇN(
©ª6£¬ÝèUVW/&†‡1âIERXô)œDq ·FB,r->, G>aIL8=,)ÂYWIE-HQ¨€BGJSNS)Uэ->QB,ˆF=I 5, G>,r8B; ,ˆ35,.-@YŽ,r, 8Œ->QB,ˆCE, 8BA9-HQBR­S)U
-HQB,­€h6)G>6)CLCE, C5R>ILF=, R (
©`3À¬ÝèU"V0/7†‡OILRWôDœ)q BÅh8BF:-HQB,­FBI u,rG>,r8B; ,­3w,r-@Ye, , 8“-HQB,­CL,r8BA)->QBR0S)U·-HQB,­€h6)G>6)CLCE, C5R>ILF=, R (
sutwvyxzf{`tw|~}
d¤<B€B€wSDR>,XVW/&†‡1›‹–ô)œDq (~‰0QB, 8“/7†‡1‘6)8BF“2X†‡Oâ6)G>,­,rפ<=ILCK6i-H, G>6)Cu->G>IK698BADCL,rR (  G>6±Y X‚
€w, GJ€5,r8BFBIE; <BCL6)G&-HS“1O(L©ªRJ, ,ˆåILAB(_'f¬PóS)-H,-HQB69- † 3BILRJ, ;.-HRXVW/&†‡1âRJS“-HQB69-XV X†‡‚ ‹
ä
)ä œ q (e¡+, 8=; , X‚”‹ †n¼DÇj‹!'D("Ýè-U`SDCLCESfYWRe-HQh69-+‚ž† ‹ † Õ ® ^‚ Õ ‹ ä=("¯¢,^6)CLRJS
S)3BR>,rGJD,­-HQB69-
/&1 ® 2jO‘‹ 1Ÿ† ® †‡Ob‹g1‚b½°‚ž† ® ©ªO‚ ® ‚ž– † ¬~‹–Ç)‚ž†gÌ
RJIL8B;r,j‚§IERT-HQB,­aILFN?è€5SDIE8£-TS)U1O(~¡+,r8B; ,j/&1 ® 2jOc‹‡Ç ä=(
D C
D C
M
M

K K P
A B
Q
O
O

A B

Fig. 2
d¤<B€B€wSDR>,[V0/7†‡O ‹™ôDœDqŸRJSž->Qh69-V0/7†g1 ‹Ð'fÇ)œ)q (  G>6Y  ->QBG>SD<=ADQ™€h6)G>6)CLCE, C"-HS
Fig. 1

/P2”©³YWIE-HQ‘SD8/P1 698BFcS)8ž1O¬.(ˆ©`R>, ,åILAB( ÇD¬PA£6)IE8† 3=ILR>,r;r->RjVW/&†‡1lR>S-HQB69-


V ^† ‹ V X † ”‹ ôDœ q (‰0Qp<BRX†âIER6)8„,rפ<=ILCK6i-H, G>6)C˜-HGJIK6)8BA)CL,)(’¡+, 8=; ,FBIL6)aˆ,.-H,rG
€w, GJ€5,r8BFBIE; <BCL6)Gˆ->S„1O 6)CER>SŒ3=ILR>,r;r->R:†(ɉ0QBILRA)IED,rRO† ‹j1(–Ý@8n->QB,-HGJIK6)8=ADCL,rR
O†‡26)8BFŒ1  ·Ye,Qh6),1 ‹bO†4˜VWO†‡2 ‹™'fÇ)œ)qX‹lVW1  6)8BFŒVO2^† ‹
V j 1 ©`€BG>S)€5,rGJ-@«žS)U0;.«¤; CLIE;פ<B6)FBG>IECK69->, G>6)C³¬Ê(¡+,r8B; ,O†‡2 6)8BF¢1  6)GJ,ˆ;rSD8BADGJ<B, 8£-XR>S
->Qh69-XO2‹1 (:‰0Q¤<=RX/&1 ® O2â‹b/ ([óS)-H,-HQh6i-X/ â‹ ‚ lRJIL8B;r,‚’/  lILR^6
€B6)GH6)CECL,rCLSDADG>6)a[(W¦Ž<=-­ ‚ cILR+-@YWIE; ,X->QB,Ÿ6)C_-HI_-H<BFB,^S)U˜-HGJIK6)8=ADCL, ^†4(­dNIE8B; , † ‹›ÇN5->QB,
69CE-HI_-H<BF=,&S)U ^† IERÇ þ 䣼)Çj‹ ä=(˜‰0QBIERWADIE), RW!/ ¿‹–Ç ä=(
²v³z±´pµi|·¶Nz±´žsutwvyxzf{`tw|~}
ÁR>IE8BA­R>SDa,0-HG>IEADSD8BS)aˆ,.-HGJ«))Ye,+; 6)8ˆA),r-"R>S)CL<=->ILSD8BR U`S)G˜35S)->Q->QB,W€h6)Gƒ-HR"R>IEa<BCE-H6)8B,rSD<BR>C_«D(*Â,.-
‚Œu• 3w,X->QB,aILFN?è€5SDIE8£-HRWS)U~/&1698BFž2^OâG>, RJ€5,r;r->IED,rCE«)(j‰0Q=, 8¨•0Ìʆg"Ì Ì͂ 6)GJ,; SDCECLIE8B,6)G
©`R>,r,ŸåILA=( ä6)8BFåILAB( mp¬.(*,r-7VW/&†‡‚ ‹¿¹B©y‹!VWO†‡•~¬.u6)8B F X† ‹ZÏh(7d¤IL8B;r,X/7†g1l6)8BF
2^†‡O 6)G>,:RJILaILCK69Gj->G>IK698BADCL,rR "IEU&†‡O ‹¸†‡2 ‹âÚn-HQ=, 8ˆg/¸‹¸†‡1 $ ‹ #¤ÚnU`S)GŸR>SDa,
€wSDRJIE-HI_D,7;rSD8BRJ-H6)8£- #5(
óS±Yn†g‚”‹ #¤Ú+; S)RÀ¹Np†‡•Þ‹—ÚP;rSDRÀ¹NDRJS&-HQh6i%- X‚â' ‹ &(#¤ÚP;rSDRÀ¹ ® )Ï &f6)8BF X•„‹¥ÚP;rSDRÀ¹À½Ïh(
+CLRJS/P‚”‹ #¤ÚPRJIL8¹Ÿ6)8BF“O•„‹¥ÚPRJIL8¹N("ÁR>IE8BA
/P‚ Õ  ½ X‚ Õ ‹g!/  Õ ‹‡*O  Õ ‹‡O• Õ  ½ X• Õ Ì
Ye,­AD,r-
# Õ Ú Õ RJIL8 Õ ¹W½g+© #¤Ú+; SDRÀ¹ ® Ϥ¬ Õ ‹¥Ú Õ RJIL8 Õ ¹W½‡©³ÚP;rSDRÀ¹W½ÃϤ¬ Õ Ä

m
L
D C
L
x x D C
x x

M kx θ M kx

A K
kx θ kx B
O
O

A B
K

d¤ILa€BCLI_Åh;69->ILSD8ADI_D, R
Fig. 3 Fig. 4

©+# Õ ® 'f¬@Ú Õ ‹gÇiÚwÏw©+#­½ 'f¬=;rSDRÀ¹ Ä


d¤IL8B;r, #^½—'jknœ==Ye,^AD,.-^©+# ® 'f¬@Ú:‹‡Ç9Ïe; S)RÀ¹N(˜‰0Qp<BR
/&1 ® 2jOc‹–ÇB©ª/P‚ ® •"O¬¸‹ Ç=©+#¤Ú+R>IE8W¹ ® ÚPRJIL8W¹D¬
‹ Ç=+© # ® 'f¬@ÚPRJIL8W¹
‹ mDÏe; S)Rh¹ R>IE8W¹
‹ Ç9ÏeR>IE8WÇ)¹ Ä
ÝèUTV0/7†‡1â‹cô)œDq -HQB,r8„Ç)¹‹‘ôDœDqr(ÝèUTV0/7†‡OЋcô)œDq -HQB,r8ŒÇ9¹:‹'fÇ9œDq (Ý@8Ö,rIE->QB, G­;69R>,
RJIL8Ç)¹j‹ 䣼DǤ(˜ÝèU˜Ï‹gÇNB-HQ=, 8[/&1 ® 2^Oc‹–Ç äNhIE8:35S)->Q-HQB,­;69R>, Rr(
ôN(+ҘGJSf),j->Qh69- õ
©ª6£¬:oÈ oW- ½ , oW- ½ . o¤M
©`3À0 ¬ /k /½ , /½ . /NM
©`;±¬ã¨k ã½ , ã½ . ãU`SDGW69CLCuIE8£-H, A), G>RW ã 1—š=(
sutwvyxzf{`tw|~}
¯Ö,Qh6),:©ðÇ Ä ÇD¬ Õ ‹¿m Ä /9m[ÈÉoNuRJS-HQh69- ok¿Ç Ä ÇN(7¡, 8B;r, . o:k Ç Ä Çk›' Ä mB 69RŸ©ƒ' Ä mp¬ Õ ‹
' Ä šDôŸÈnÇ Ä Ç¤(˜‰0QB, GJ,rU`SDGJ, , ok . ok–' Ä mB(%PFBF=IL8BABNYŽ,jAD,r-
o½ , o0½ . oknÇ Ä Ç½—' Ä mP½—' Ä mX‹‡o Ä
¯Ö,­SD3BRJ, GJ),j->Qh69- äÈËä=2 , /^‹‡Ç698B F . /È , /^‹‡ÇN(˜‰0Qp<BR
/½ , /½ . /Èn䏽ËÇW½ËÇ­‹ 34 È /Ä
d¤<B€B€wSDR>,Ÿ5 ã 1Zš=(7‰0QB,r8¨ã5Õ 1¿š)㝍 R>S->Qh69-75 ã 1Zä ã(7‰0QBIER&ADIE), R 7 ã 6¿ã ¼)ä=(7‰0QB,rG>,rU`S)G>,
ã. žÈ , ãžÈ  ã 6¥ã¼)ä=( ¯Ö,7-HQp<BRSD3=-H6)IL8
ぽ , ã½ . ãžÈÉ©ªã ¼)䣬½g©ªã¼9䣬½‡©`ã¼)䣬˜‹ ã Ä

879 œ 879

o
    !"# $&%

')(+*,.-0/&132546,&7)897): ;<-=,.>?7)8A@CBD, E-=F=GH7)8A@CBD,)IJBH,.-0KLNMO46,+-=PA,RQGHES>?T6UCGH8V-NW"U)X12YZ/&1!F[, W[T\,]:.>


-=GD^), BD_)(`*,]-0-=PA,aT6, F=T6, 8<EAGH:];ABb7)FcXdF=UCQeMO-NU/R2!7)8AE9-=PA,aT6, F=T6, 8<EAGH:];ABb7)Ff7g-h1!-=U12iQ, ,.-
GD8kjl(cm`F=Un^C,o-NPA7g-+jKGHWh, pJ;q7gBr-NUs-NPA,o:]GHF=:];AQs>?FN7gEAGH;AWfU)X/R12( tu'wvwx
y{z6|~}w€dz6c‚ *,]-„ƒ 46, -NPA,†:]GHF[: ;AQs>
:], 8V-NF[,3U)X/&12(ˆ‡)UCGD8ƒ‰KYŠƒ3j‹7)8AE
ƒ‰M(ŒŽ, W[PAUn-=Pq7g-‘13K9ƒ3j GHW7 A
F[, :.-’7)8A@)BH,)(“”-•XdUCBHBDUw–W-=Pq7g-K—j ˜
13ƒ ˜ ™Yš-=PA,!:]GHF[: ;AQs>?F=7)EAGH;<W›U)X
K

/R12(
œ 4AW[, F^C,-NPq7g-‰ž–j132Ÿ˜ ž–j¡2˜
¢)£C¤ ( ¥ˆ, 8A:],¦12§¡j GHW¨7e:]_S: BDGH:
F

pJ;q7gEAF=GDBb7g-=, FN7gB©( ªhPJ;AW¦ž–¡j1 ˜ N O

'«C£ ¤k¬ ž–12o/3( ­f;Z-®ž–1ƒ‰/ ˜


¯)žˆ12o/ 7)8<E ƒ‰M 4AGDW=,]:]-NW°ž–1ƒ‰/‰( B
¥ˆ, 8A:],ž–1ƒ3M˜žˆ132§/‰(—ŒŽ,-NPJ;AW D C
U)4<-’7)GD8
ž–M§j1›±²ž–1ƒ3M³˜´žˆ¡j•1³±°žˆ132‰¡µ˜5'n«C£ ¤ ¶
ªhP<GHWoGHQTABHGD, Wˆ-NPq7g-‰1—LNƒL’MhL’j·7gF=,: UC8S>?:]_S:]BHGH:g(¥+, 8<: ,—žˆ1M§ƒŸ˜ žˆ1j•ƒ(­f;<-§UC4<W=, F^C,
-=Pq7g-0žˆ13M‰ƒO˜¨¢C£ ¤ W[GH8A:],&ƒ3M´GDW¸T6, F[T\,]8AEAGD: ;ABH7)Fc-=U3/R1—(`ªhPJ;AW–ž–1jƒO˜³¢)£ ¤ ("¹ZGD8A: ,Rj1
7g8AEºƒ3K7)F=,aT6, F[T\,]8AEAGD: ;ABH7)F0-NU12YqGD-hXdUCBDBHUn–Wf-=Pq7g-ˆ1K9ƒ‰j»GHWˆ7sF=,]:]-’7g8A@CBH,g(
¼|½n¾J¿ÀÂÁZn¾ yrz6|Ã}Äw€dz6c‚ Œl,s:7)8„7)BDW=U@C,.-&-NP<,s: UC8A:]BH;AW[GHUC8;<W=GH8<@9-NF=GD@CUC8AU)Q,.-NF[_)( œ 4<W=, F^C,
-=Pq7g-žˆj•M‰1‹˜žh/&M§¡ ˜Ÿ¢C£ ¤h¬ žh/3Ic7)8AEšž–1j•Mµ˜»'n«C£ ¤–¬ ž–1µW[GH8A:],912§¡j GHW‰7
:._S: BHGD:apJ;q7)EAF[GHBb7À-N, F=7)B©(Å+W[GH8A@s-NP<,§W[GH8A,Æ>?F[;ABH,aGD8k-=PA,a-NF[Gb7)8A@)BH,&13M‰j„Y
j1 ˜ 1M ¶
W[GH8ˆž–jM§1 W[GH8ˆž–1M§j
ªhP<GHWhF=,]EA;A:], W+-=U
j1i˜È¯Ç :]W[GHUC8–Wq2 / ˜³™: UCWŠ/ ¶
­¸;<-ˆ1K ˜›ÉSÊC¯§˜›™‘W=GH8h/‰("ªhPS;<W
j•K—Ëf˜³j13˱°13K—Ë0˜›™&Ë ¶
ªhP<GHWh@CGÌ^C, Wˆj•K͘³™(
¯S(+m`F[Uw^),§-=Pq7g-h-=PA, F[,‰,ÆÎ<GDW[-–-?¸UGD8<Ïq8AGÌ-N,aW=,]pJ;A, 8A:], WЩÉCÑVҝÑÀÓqÔh7g8AE Ð~ՒÑCҝÑÀÓqÔfUgXT6UCW=GÌ-NGÌ^C,aGH8V-=, @C,]F=W
W[;A:’P•-=Pq7g-h-=PA,aXdUCBHBDUw–GD8A@:]UC8AEAGÌ-NGHU)8AWhPAUCBDEW=GHQ3;ABD-N7)8A, U);AW=BÌ_\Ö
× GbØ'‰Ù²ÉAÔ0ÙÚÉ Ë ÙÛÉVÜ+Ù´ÝnÝnÝ]I
× GHGbØ9ÉVÑÙÚՒÑÙ²É ÑË Y<XdUCF–7gBHBrޑ߮'CI
× GDGHG½ØÉVÑ ¬ '‰EAGÌ^SGHEA,]W–Õ’Ñ ¬ 'CYAXdUCF–7)BDB{ޑ߮'CI
× GD^<ØÉ ÑË ¬ '‰EAGÌ^SGHEA,]W–Õ ÑË ¬ 'CYAXdUCF–7)BDB{ޑ߮'C(
ut 'n¢Àx
y{z6|~}w€dz6c‚ *,]-+;AW&BHUSUCà7À-+-=PA,3TAF[UC4ABD, QŸU)X ÏA8AEAGH8<@-?¸U9T\U)W=GD-=GD^),‰GD8J-=, @C,]F=WRɊLNÕoW[;A:’Pá-NPA7g-
'oÙÛÉÙÚÕRÙÛÉ Ë YŠÉ ¬ 'oEAGD^SGHE<, WˆÕ ¬ '§7)8AE•É Ë ¬ '‰EAGÌ^SGHEA,]W–Õ Ë ¬ 'C(`ªhPJ;AW–f,§Pq7^C,
Õ ¬ 'R˜³â × É ¬ 'wØ.L 7)8AE Õ Ë¸¬ '&˜¨ã × É Ë¸¬ 'wØ ¶
ä BDGHQGH8q7g-=GH8A@‰Õ+XdF[UCQ -=PA, W[,‰,]pS;A7g-NGDUC8AW YA¸,§@C,]-
× â ˸¬ ãÃ؝És˜´â Ë"¬ ¯Câo±°ã ¶
ªhPJ;AWˆGD-fXdUCBHBDUw–W0-=Pq7g-
ɘ â Ë â ¬ Ë ¯C¬ âoã ±°ã ˜i' ¬ ¯ â × â Ë ¬¬ ããÃØ ¶
Œl,R8<, , EkÉ-=U4\,R7)89GD8JÜ -=, @C,]F (cåfPAUSUCW[,oâ Ë ¬ 㘳¯§W=U3-=Pq7g-hɘi'±lã ¬ ☛â Ë ¬ â ¬ '&7)8AE
Õ&˜¦â × É ¬ 'wر®'3˜æâ ¬ â Ë ¬ ¯C≱³'C(&Œl,07)8V-§Éç!'–PAGH:’PGDWa7)W=W[;AF=,]E‘GDX`¸,s:’PAUZU)W=,
â‘ß!èZ(3éˆUw!ÉÙOՉGDW&, pJ;AGÌ^)7gBH, 8V-a-=U × â Ë ¬ 'wØ × â ¬ ¯CØaçi£—Ü –P<GH:’P„7)@V7)GD8áGHW&7)W=W[;AF=,]ElUC8<: ,
âßOè<(a“”-aGDWR, 7)W[_•-NUW=,],3-NPA7g-oÕÙ®É Ë GHW+, pJ;AGÌ^)7gBH, 8V-&-=Ukâ × â ¬ èVâ Ë ±ÚêJØRç´£7)8AEá-NP<GHWRGDW
7gBHW=U-=F=;A,aXdUCF–7gBHBÄâßëè<(`ªhPJ;AW–f,§EA,]ÏA8A,
ÉVÑ ˜ × Þ±²¯)Ø Ë"¬ × Þ±°¯CØ ¬ '+˜›Þ Ë ±°è)Þ±ë'CL
ÕNÑ ˜ × Þ±²¯)Ø Ü ¬ × Þ±°¯CØ Ëc¬ ¯ × Þ±²¯Cرë'&˜ëÞ Ü ±²vgÞ Ë ±íì)Þ±¨'CL
XdU)Fhބ߮')(`ªhPA, 8ºf,oW=,],§-=Pq7g-
'§ÙÚÉCÑÙÛÕNÑÙ²Õ ÑË L
XdU)F–7)BHB\ބ߮')(cî•UCF=,]Un^C, F
ÉVÑ ¬ '&˜ëÞ × Þ±íèVØ.L ÕNÑ ¬ '+˜›Þ × Þ±°èVØ × Þ±°¯CØ
7g8AE
É ÑË ¬ 'R˜¨Þ × Þ±°èVØ × Þ±ë'wØ × Þ±²¯CØÆL Õ ÑË ¬ 'R˜¨Þ × Þ±°èCØ × Þ±°¯CØ × Þ±¨'wØ × Þ Ë ±šêCÞ±²¯)Ø ¶
ªhPJ;AW–¸,§PA7n^),37Tq7)GDF0U)XïEA, W[GHF[, EºW=, pJ;A,]8A: ,]WЩÉCÑCҖ7g8AE Ð~ՒÑCÒ](
èZ(R¹S;AÜTAT6UCW=,§É97)8<EÕ&7gF=,aF=, 7)B8J;AQ46, F[WˆW[;A:’P•-=Pq7g-–-NP<,§F[UZUg-NWhU)XÂ-NPA,o:];A4AGH:o,]pS;A7g-NGDUC8
ÉCð ¬ ð Ë ±íÕNð ¬ 'R˜›£7)F[,‰7gBHBrT6UCW=GÌ-NGÌ^C,&F[,7)B8J;AQ46, F[W (¸m`F=Un^C,o-NPq7À-Ö
× Gbب£sÙÛè)ÉSÕ+ñ®' 7)8<E × GHGbØëÕ+ß®ò èZ(
tu'n¢Àx
y{z6|~}w€dz6c‚ *Ä,.-+ó¸YAô"Y<õ46,&-NP<,§F[UZUg-NW–U)XÂ-NPA,o@CGÌ^C, 8k,]pS;A7g-NGDUC8r(`ŒŽ,oPq7^),
ó±šô±†õº˜ É' L óïô—±íôõ±†õ{ó‘˜ ÉÕ L óÂôÄõ˜ É' ¶
“”-hXdU)BHBHUn–W0-=Pq7g-ˆÉqL’Õ&7)F[,aT\U)W=GD-=GD^),)( Œl,o-=PJ;AW+U)4<-’7)GD8
èCÕ ˜¨è × óÂô9±šôõ±šõ\óØhñ × ó±íô9±šõ{؝Ëf˜ ' L
É ÉË

¯
–P<GH:’Pº@CGD^), Wˆ£sÙÛèCÉJÕRñ®'C(cî•U)F=, Un^), F
Õ Ë ˜ × óïô9±íôõ±šõ\ó ØöË
ÉË
˜ ó Ë ô Ë ±íô Ë õ Ë ±šõ Ë ó Ë ±²¯góïôõ × ó±šô±šõ{Ø
˜ ó Ë ô Ë ±íô Ë õ Ë ±šõ Ë ó Ë ± É ¯ Ë ¶
ªhPJ;AW
Õ Ë ¬ ¯ ˜³ó Ë ô Ë ±íô Ë õ Ë ±íõ Ë ó Ë ß ' × óÂô9±šôõ±šõ\óØ Ë ˜ Õ Ë ¶
ÉË è èCÉ Ë
ªhP<GHWhGHQTABDGH, W¸-NPq7g-ˆè × Õ Ë ¬ ¯CØ0ßëÕ Ë UCF–Õ Ë ßëèZ("¥+, 8<: ,‰Õ&ß³ò èZYA-=PA,o: UC8<: BH;<W=GHU)8kXdU)BHBHUn–W](
ê<(+÷ïGH8AEá-=PA,8S;<Q4\,]FaU)Xf7)BHBìÀ>?EAGD@CGD-R8q7g-=;AFN7)B8S;<Q4\,]F=WoW=;<:’P„-=Pq7g-a-NP<,W[;AQ»U)Xc-NP<, GHFREAGH@CGÌ-NW&GDW
'£7g8AEk,7):’P•U)Xï-=PA,oEAGH@)GD-NW0£<YD')Yø¯ZYùèsUZ:]: ;AF[Wˆ7g-ˆBH,7gW[-–UC8A:],oGH8-NPA,]Q•( tu'êgx
y{z6|~}w€dz6c‚ Œl,§UC4AW=,]F[^),§-=Pq7g-+£R±ë'¸±Ú¯–±°è˜›ì<(f¥ˆ, 8A:],‰-=PA,oF=, Q7)GD8AGH8A@-?¸U—E<GH@CGÌ-NWhQ;AW-
7g: : U);A8V-ˆXdU)F–XdU)F0-NPA,oW=;<QŸêA("ªhPAGHW–GHW–T6UCW=W[GH4ABD,&–GD-=P•ê3˜³£ˆ±íê3˜i'¸±²è‰˜´¯–±²¯Z("ªhPJ;AW+¸,
W[, ,-NPq7À-§-=PA,EAGH@)GD-NWaGD8„7)8V_‘W=;A:’PŽìÀ>?EAGD@CGD-R8J;AQ46, F§Q;AW[-§46,sXdF=UCQ»U)8A,U)Xf-=PA,: UCBDBH, :.-NGDUC8AW Ö
ú £ZL'CLƯZLNè<L’£<LNêZûJY ú £ZL'CLƯZLNè<L'CL’èSûaUCF ú £<L'CL’¯ZL’è<LƯSLƯSûJ(
åfU)8AW=GDEA, F3-=PA,9: 7)W=,GD8Ž–PAGH:’P -=PA,9EAGD@CGD-=W37gF=,—XdF[UCQ-NPA,:]UCBHBD, :]-=GHUC8 ú £<L')LƯZL’è<LN£<LNê<ûJ(•¥+,]F=,
£lUZ:]: ;AF[W—-?–GH:],‘7)8AE²-=PA,áEAGD@CGD-=W'CYü¯ZYùè<YùêlUZ:]: ;AF—UC8A: ,, 7):’Pr(O­f;Z-k£Ž:7g8A8AU)-46,-=PA,ÏqF[W[-
E<GH@CGÌ-( ¥ˆ, 8A:],&-NPA,&ÏAF=W[-0EAGH@CGÌ-fQ3;AW[-h46,aUC8A,RU)X"'CYü¯ZYüèZYùêA(c¹S;ATAT6UCW=,&¸,&Ï<Î'&7)W0-NPA,+ÏqF=W-–EAGH@CGÌ-(
ªhP<, 8-=PA,38J;AQ46, F+U)XcìÀ>”EAGH@CGÌ-–8J;AQ46, F=W+GH8º–PAGH:’P-=PA,3F[, Q7)GH8<GH8A@vEAGD@CGD-=Wˆ7)F=,‰£<Yù£<Yø¯SYüè<Yýê—GDW
vSþøÊC¯Sþ˜ŸìC£Z(á¹<7gQ,GHWo-NPA,:7)W[,—–GD-=PŽU)-=PA, F‰EAGH@)GD-NW]Ö¯ZYùè<YýêA(kªhPS;<W-NPA,8S;<Q4\,]F3U)XhìÀ>?EAGD@CGD-
8J;AQ34\,]F=W–GH8–PAGD:’Pº-=PA,oEAGH@CGÌ-NWh£ZYD'CYü¯ZYùè<Yù£<YùêUS: :];AF–GHW–ìC£sÿáꉘ´¯gêC£<(
¹S;ATAT6UCW=,-=PA,E<GH@CGÌ-NW7)F=,ºXdF=UCQ -=PA,:]UCBHBD, :]-=GHUC8 ú £<L')LƯZL’è<L 'CL’èZûJ(ÛªhPA,8S;<Q4\,]F—U)XaìÀ>?EAGD@CGD-
8J;AQ34\,]F=W§4\,]@CGH8A8<GH8A@k–GD-=P°'GHW‰vZþüÊC¯Zþ˜ìC£<(—ªhPA,8J;AQ34\,]F3U)X¸-NPAUCW[,—46, @CGD8A8AGD8A@–GD-NP ¯kGDW
vSþøÊ × ¯ZþüØ × ¯Sþø؈˜eè)£º7g8AE-NPA,8J;AQ46, FaU)Xc-NP<UCW=,46, @CGD8A8AGH8<@9–GD-NP‘èGDW§vZþøÊ)¯Zþr˜ ìC£<(3ªhPS;<Wo-=PA,
-=U)-’7gBˆ8J;AQ34\,]F—GH8š-NP<GHW:7)W[,áGHWìC£s±®èC£s±®ìC£í˜ 'nv)£<(  BÌ-N, F[8q7g-=, BD_)Y¸¸,á:7g8Ú7)BDW=UŽ:]UC;A8V-
GÌ-º7)WkXdUCBHBDUw–W]ֆ-NPA,„8J;AQ46, FºU)XsìÀ>”EAGH@)GD-8J;AQ46, F[W•U)8A,l: 7)8³U)4<-’7)GD8ÛXdF=UCQ -=PA,l:]UCBHBD, :.-NGHU)8
ú £ZL'CLƯZLNè<L'CL’èSûs–GD-NPŽ£7)BDW=U•7)W‰7•T6UCW=W[GH4ABD,ÏAF=W[-3EAGH@)GD-oGHW‰ì<þüÊ × ¯SþøØ × ¯ZþüØo˜'n«C£<Iï-NPA,8J;AQ46, F
UgXˆìÀ>?EAGD@CGD-§8J;AQ46, F[WsUC8A,9: 7)8†UC4<-N7)GH8„XdF=UCQ‹-NP<,9: UCBDBH,]:]-NGDUC8 ú £ZL'CLƯZLNè<L'CL’èSû—GD8Ž–PAGH:’Pí£GDW
-=PA,&ÏqF[W[-–EAGD@CGD-fGHWˆvZþüÊ × ¯SþøØ × ¯ZþüØ`˜›èC£Z(`ªhPS;<W–-=PA,o8J;AQ46, FhU)XïìÀ>”EAGH@CGÌ-08J;AQ46, F[W0XdUCF=Q, E4V_9-=PA,
:]UCBHBD, :.-NGHU)8 ú £<L'CL’¯ZL’è<L')L’èZûoW=;<:’P•-NPA7g-–8AU8J;AQ46, F–Pq7)W–GD-=W0ÏqF=W-ˆEAGH@CGÌ-–£sGHWa'n«)£ ¬ è)£‰˜O'wv)£<(
÷ïGH8q7)BDBD_BHUSUCà—7g-h-NPA,§: U)BHBH,]:]-=GHUC8 ú £<L 'CLƯZL’èZLƯZLƯSûJ(¥+,]F=,o-NPA,o8J;AQ46, FˆU)XïU)X`ìÀ>”EAGH@)GD-08J;AQ34\,]F=W
GD8ᏖPAGH:’Pí'GHW&-=PA,sÏqF=W-oEAGH@CGÌ-aGHW§vSþøÊ)èZþr˜ ¯)£<IÄ-NP<,s8S;<Q4\,]FoU)X¸-=PAUCW[,PA7n^SGD8A@•¯k7gWo-=PA,ÏqF[W[-
E<GH@CGÌ-–GHWRvZþøÊ)¯ZþA˜®ìC£ZI\7g8AE-NP<,‰8J;AQ46, F+U)X -=PAUCW=,§Pq7^SGH8A@—è—7)Wˆ-NPA,oÏqF[W[-&E<GH@CGÌ-–GHWRvZþøÊgè<þA˜O¯)£Z(
ªhPJ;AW¸-NPA,+8S;<Q4\,]FfU)XÂ7gEAQGDW=W[GH4ABD,–ìÀ>?EAGD@CGD-"8J;AQ34\,]F=W0PA,]F=,&GDW0¯g£"±„ìC£"±„¯)£‰˜5'n£)£<( ªhPAGDWfQ7n_
7gBHW=U‰4\,RUC4<-’7gGH8A,]E—;AW[GH8A@3-=PA,&U)-=PA, F¸Q,.-NPAUSE9U)XÄ:]UC;A8V-NGD8A@AÖ`ìZþøÊ)èZþ ¬ vZþüÊ)è<þJ˜i'n¯)£ ¬ ¯)£§˜5'n£C£Z(
÷ïGH8q7)BDBD_+-NPA,f-=U)-’7gBZ8J;AQ46, FU)X6ìÀ>?E<GH@CGÌ-ï8J;AQ34\,]F=W GD83–PAGH:’Ps, 7):’PsU)Xq-=PA,0EAGD@CGD-=W£<YÌ'CYü¯ZYùèR7)TAT6,7gF=W
7À-–BH,7gW[-–UC8A:],oGHWˆ¯gêV£ˆ±ë'wv)£–±¨'£C£§˜›êV¢C£<(
vS(ˆªhP<F=, ,f8AUC8 ], F=URF=,7gBZ8J;AQ46, F[W`ÉqL’ÕnL Ç 7)F=,¸WN7)GDE‰-NUR4\,¸GH83PA7)F=QUC8AGD:¸TAF[UC@CF[, W=W[GHUC8‰GDX É' ± ' ˜ ¯Õ (
÷ïGH8AE37)BHBV-NP<F=, ,Æ>”-=, F=Q5Pq7)F[QU)8AGH:"TAF[UC@CF=,]W=W[GHUC8AW ɊLNÕwL Ç U)XAW[-NF[GH:.-NBD_§GD8A: F[,7)W[GH8A@+T\UCW[GD-=GD^C,cGH8VÇ -=, @C,]F=W
GD89–PAGH:’Pɘ´¯g£7g8AEÕ+EAGD^SGHE<, W Ç ( tu'wx
y{z6|~}w€dz6c‚ ¹ZGH8A:],‰¯)£<L’ÕnL Ç 7gF=,aGH8kPq7gF=QUC8AGH:RTAF=UC@)F=, W[W=GDUC8rYA¸,§PA7n^),
' ± ' ˜ ¯L
¯)£ Ç Õ
è
–P<GH:’PºF=, E<;A: ,]Wˆ-=UÕ Ç ±°¯)£CÕ ¬ Cê £ Ç ³ ˜ £<(`ªhP<GHWhQ7n_7)BHW[Us4\,a–F=GÌ-=-=, 8ºGH8-NPA,aXdUCF[Q
× êV£ ¬ Õ.Ø × Ç ±²¯)£CØ`˜³«C£C£ ¶
ªhPJ;AWa¸,Q;AW[-&Pq7^C,¯)£ÙOÕsÙOêV£9UCF]Yr, pJ;AGÌ^)7gBH, 8V-=BD_CYr£Ù®êV£ ¬ ÕsÙ!¯g£<(3*,.-o;AWa: UC8<W=GHE<, F
-=PA,aXÃ7):]-=UCF=GDWN7g-=GHUC8—U)X «)£C£sGH8–PAGH:’PºUC8A,a-=, F=QÍGHWhBH,]W=Wh-NPA7)8•¯)£<Ö
× êV£ ¬ Õ.Ø × Ç ±°¯)£VØ"˜›«C£)£‰˜5'‰ÿ«C£C£§˜´¯sÿêV£C£§˜›êÿ¯g£C£
˜´vÿ 'ìC£§˜³«ÿŽ'n£C£§˜i'n£sÿ«)£‰˜5'nìsÿ‘v)£ ¶
Œl,a-NPJ;AW–@C,.-ˆ-=PA,oTq7)GDF=W
× ÕnL Ç Ø"˜ × èC¢<L )«)£VØ.L × èC«<L’è)«C£VØ.L × èCì<L 'n«C£VØÆL × èVvSL'êC£CØ.L × èV¯SL’«)£VØ.L × èC£<L=ìC£VØÆL × ¯wêAL’èC£)Ø ¶
 QU)8A@§-=PA, W[ , aTq7)GDF=W YV¸,RW[, ,+-=Pq7g-cUC8ABÌ_voTA7)GHF[W × èC¢<L g«C£VØ.Y × èC«<L’è)«C£VØ.Y × èCì<L'«C£VØ.Y × èVvZL' êV£VØ.Y
× èC£<L’ì)£VØXd;ABÌÏqBHBR-=PA,„: UC8AE<GD-NGDUC8ëU)X‰EAGD^SGHW[GH4AGDBHGÌ-?_\Ö ÕEAGD^SGDEA, W Ç (ͪhPJ;AW•-=PA, F[,Ž7gF=,lv†-=F=GDTABH,]W
W=7g-NGDW[X½_SGH8<@-NPA,oF=,]pJ;AGHF[, Q, 8V-ˆU)Xï-=PA,oTAF=U)4ABH,]Q•(
ìZ(+÷ïGH8AE-NP<,38J;AQ46, FˆU)X`7gBHBrGD8J-=, @C,]F>”W=GHE<, E      "!o-=F=Gb7g8A@CBH,]W0–GD-=PT6, F=GDQ,.-N,]F
¯g£C£C«<( tu'nìÀx
y{z6|~}w€dz6c‚ *,]--NP<,W=GDEA, W46,ºðÄL=ðÄ"L #\Yc–PA, F[,•ðÄ$L #š7)F[,•T6UCW[GD-NGÌ^C,kGH8V-=, @C,]F=W (¨¹SGH8A:],º¸,á7)F[,
BDUZU)àSGD8A@9XdUCF§UC4<-=;AW=,Æ>ö7)8A@)BH, E‘-NF[Gb7)8<@CBH,]W %Y #†ç!ðÄ(ºî•UCF[, Un^C,]F Y"¯wð& ± #‘˜Ÿ¯)£C£)«ºW[PAUn–W‰-NPA7g- #
GDWo,]^C,]8r(k­f;<'- # Ùið±ÚðÄYï4V_‘-NF[Gb7)8<@CBH,GD8A, pJ;q7gBHGD-?_)(ªhPJ;A(W # Ù'n£)£)êA(ªhPS;<W‰-NP<,T6UCW=W[GH4ABD,
-=F=GDTABH,]Wï7gF=, × #6L=ðÄL=ð\Ø`˜ × 'n£C£V¯ZL’v)£Cè<LÆvg£CèVØ.Y × 'n£C£C£ZLÆv)£)êAL’v)£)êJØÆY × ¢C¢C«ZLÆv)£VvZL’v)£VvCØÆYw7)8AEW[URUC8r(ïªhPA,
@), 8A,]FN7)B XdUCF[QµGDW × #6L=ðÄL=ð\Ø&˜ × 'n£)£)ê ¬ ¯)â\LÆv)£V¯+±›â6LÆv)£V¯&±ëâAØÆY–PA, F[,9â˜'CLƯZL’èZL ¶¶¶ LÆv)£<'C(
­¸;<-h-NPA,o:]UC8AEAGÌ-NGHU)8k-=Pq7g-h-=PA,a-NF=GH7)8A@CBD,&GHWhU)4<-N;AW[,oBH,7gEAWh-NU
× 'n£C£gê ¬ ¯CâAØ Ë çë¯ × v)£V¯h±°âAØ Ë ¶
ªhP<GHWhW=GDQT<BHGDÏA, Wf-=U
v)£C¯ Ë ±²â Ëc¬ ì × v)£V¯CØ[âçÚ£ ¶
¹SUCBD^SGH8<@-NPAGDWhpS;A7)EAFN7À-NGH:aGD8A, pJ;q7gBHGD-?_XdUCFˆâ\YA¸,oW=, ,o-NPA7g-
âÙÛv)£C¯ × è ¬ ¯ ò ¯CØ.L UCF âçëv)£C¯ × è–±°¯ ò ¯gØ ¶
¹SGH8A:],aâñ¨v)£<'CYJf,a: 7)89F[;ABH,+UC;<-f-=PA,&W[, : U)8AEkT\UCW[W=GD4AGHBDGD-?_C(ªhPJ;AWˆâÙÛv)£V¯ × è ¬ ¯ ò ¯gØ.YS–PAGH:’P
GDW–7)TATAF[UÎ<GDQ—7À-N, BÌ_9«)ì<(D' êVèV¯Z(Œl,o: UC8<: BH;<EA,a-NPq7g-Râ9ñë«Cì<(`ªhPJ;AWˆf,o@C,.-+«Cì-NF=GH7)8A@CBD, W
× #6L=ðÄL=ð\Ø`˜ × 'n£C£gê ¬ ¯Câ\LÆv)£C¯h±²â\LÆvg£V¯h±²âAØÆL â˜i')LƯZL’è<L ¶¶¶ LN«Cì ¶
ªhP<,&Bb7)W-0UC4Z-N;AW[,R-NF[Gb7)8A@)BH,ˆGH8-=PAGHW¸BHGDW[-¸GHW Ö × «CèV¯SYøv)«)«<Yøvg«C«VØ.( × “”-fGHWf, 7)W[_-NU:’P<, :’à—-=Pq7g-h«)èV¯ Ë ¬
vg«C« Ë ¬ v)«C« Ë ) ˜ )èCìsçÛ£<Y<–P<, F=,‰7)Wˆ«CèC£ Ë ¬ v)«C¢ Ë ¬ v)«C¢ Ë ˜ ¬ êV¢gêJ¯ÙÛ£<(üØ

*,+ £ ,
* +

ê
Regional Mathematical Olympiad-2009
Problems and Solutions

1. Let ABC be a triangle in which AB = AC and let I be its in-centre. Suppose


BC = AB + AI. Find ∠BAC.
Solution:

A B

We observe that ∠AIB = 90◦ + (C/2). Extend CA to D such that AD = AI.


Then CD = CB by the hypothesis. Hence ∠CDB = ∠CBD = 90◦ − (C/2). Thus

∠AIB + ∠ADB = 90◦ + (C/2) + 90◦ − (C/2) = 180◦ .

Hence ADBI is a cyclic quadrilateral. This implies that


B
∠ADI = ∠ABI = .
2
But ADI is isosceles, since AD = AI. This gives

∠DAI = 180◦ − 2(∠ADI) = 180◦ − B.

Thus ∠CAI = B and this gives A = 2B. Since C = B, we obtain 4B = 180◦ and
hence B = 45◦ . We thus get A = 2B = 90◦ .

2. Show that there is no integer a such that a2 − 3a − 19 is divisible by 289.


Solution: We write

a2 − 3a − 19 = a2 − 3a − 70 + 51 = (a − 10)(a + 7) + 51.

Suppose 289 divides a2 − 3a − 19 for some integer a. Then 17 divides it and hence
17 divides (a − 10)(a + 7). Since 17 is a prime, it must divide (a − 10) or (a + 7).
But (a + 7) − (a − 10) = 17. Hence whenever 17 divides one of (a − 10) and (a + 7),
it must divide the other also. Thus 172 = 289 divides (a − 10)(a + 7). It follows
that 289 divides 51, which is impossible. Thus, there is no integer a for which 289
divides a2 − 3a − 19.
3. Show that 32008 + 42009 can be wriiten as product of two positive integers each of
which is larger than 2009182 .
Solution: We use the standard factorisation:

x4 + 4y 4 = (x2 + 2xy + 2y 2)(x2 − 2xy + 2y 2 ).

We observe that for any integers x, y,

x2 + 2xy + 2y 2 = (x + y)2 + y 2 ≥ y 2 ,

and
x2 − 2xy + 2y 2 = (x − y)2 + y 2 ≥ y 2.
We write 4 4
32008 + 42009 = 32008 + 4 42008 ) = 3502 + 4 4502 .
Taking x = 3502 and y = 4502 , we se that 32008 + 42009 = ab, where
2 2
a ≥ 4502 , b ≥ 4502 .

But we have 2 182


4502 = 22008 > 22002 = 211 > (2009)182 ,
since 211 = 2048 > 2009.

4. Find the sum of all 3-digit natural numbers which contain at least one odd digit
and at least one even digit.
Solution: Let X denote the set of all 3-digit natural numbers; let O be those
numbers in X having only odd digits; and E be those numbers in X having only
even digits. Then X \ (O ∪ E) is the set of all 3-digit natural numbers having at
least one odd digit and at least one even digit.The desired sum is therefore
X X X
x− y− z.
x∈X y∈O z∈E

It is easy to compute the first sum;

X 999
X 99
X
x = j− k
x∈X j=1 k=1
999 × 1000 99 × 100
= −
2 2
= 50 × 9891 = 494550.

Consider the set O. Each number in O has its digits from the set {1, 3, 5, 7, 9}.
Suppose the digit in unit’s place is 1. We can fill the digit in ten’s place in 5 ways
and the digit in hundred’s place in 5 ways. Thus there are 25 numbers in the set
O each of which has 1 in its unit’s place. Similarly, there are 25 numbers whose
digit in unit’s place is 3; 25 having its digit in unit’s place as 5; 25 with 7 and 25
with 9. Thus the sum of the digits in unit’s place of all the numbers in O is

25(1 + 3 + 5 + 7 + 9) = 25 × 25 = 625.

2
A similar argument shows that the sum of digits in ten’s place of all the numbers in
O is 625 and that in hundred’s place is also 625. Thus the sum of all the numbers
in O is
625(102 + 10 + 1) = 625 × 111 = 69375.
Consider the set E. The digits of numbers in E are from the set {0, 2, 4, 6, 8}, but
the digit in hundred’s place is never 0. Suppose the digit in unit’s place is 0. There
are 4 × 5 = 20 such numbers. Similarly, 20 numbers each having digits 2,4,6,8 in
their unit’s place. Thus the sum of the digits in unit’s place of all the numbers in
E is
20(0 + 2 + 4 + 6 + 8) = 20 × 20 = 400.
A similar reasoning shows that the sum of the digits in ten’s place of all the numbers
in E is 400, but the sum of the digits in hundred’s place of all the numbers in E
is 25 × 20 = 500. Thus the sum of all the numbers in E is

500 × 102 + 400 × 10 + 400 = 54400.

The required sum is

494550 − 69375 − 54400 = 370775.

5. A convex polygon Γ is such that the distance between any two vertices of Γ does
not exceed 1.

(i) Prove that the distance between any two points on the boundary of Γ does
not exceed 1.
(ii) If X and Y are two distinct points inside Γ, prove that there exists a point
Z on the boundary of Γ such that XZ + Y Z ≤ 1.

Solution:

(i) Let S and T be two points on the boundary of Γ, with S lying on the side
AB and T lying on the side P Q of Γ. (See Fig. 1.) Join T A, T B, T S. Now
ST lies between T A and T B in triangle T AB. One of ∠AST and ∠BST is
at least 90◦ , say ∠AST ≥ 90◦ . Hence AT ≥ T S. But AT lies inside triangle
AP Q and one of ∠AT P and ∠AT Q is at least 90◦ , say ∠AT P ≥ 90◦ . Then
AP ≥ AT . Thus we get T S ≤ AT ≤ AP ≤ 1.
Q F
T Z2
P E

A S B C Z1 D

Fig. 2
Fig. 1

3
(ii) Let X and Y be points in the interior Γ. Join XY and produce them on
either side to meet the sides CD and EF of Γ at Z1 and Z2 respectively. WE
have

(XZ1 + Y Z1 ) + (XZ2 + Y Z2 ) = (XZ1 + XZ2 ) + (Y Z1 + Y Z2 )


= 2Z1 Z2 ≤ 2,

by the first part. Therefore one of the sums XZ1 + Y Z1 and XZ2 + Y Z2 is
at most 1. We may choose Z accordingly as Z1 or Z2 .

6. In a book with page numbers from 1 to 100, some pages are torn off. The sum of
the numbers on the remaining pages is 4949. How many pages are torn off?
Solution: Suppose r pages of the book are torn off. Note that the page numbers
on both the sides of a page are of the form 2k − 1 and 2k, and their sum is 4k − 1.
The sum of the numbers on the torn pages must be of the form

4k1 − 1 + 4k2 − 1 + · · · + 4kr − 1 = 4(k1 + k2 + · · · + kr ) − r.

The sum of the numbers of all the pages in the untorn book is

1 + 2 + 3 + · · · + 100 = 5050.

Hence the sum of the numbers on the torn pages is

5050 − 4949 = 101.

We therefore have
4(k1 + k2 + · · · + kr ) − r = 101.
This shows that r ≡ 3 (mod 4). Thus r = 4l + 3 for some l ≥ 0.
Suppose r ≥ 7, and suppose k1 < k2 < k3 < · · · < kr . Then we see that

4(k1 + k2 + · · · + kr ) − r ≥ 4(k1 + k2 + · · · + k7 ) − 7
≥ 4(1 + 2 + · · · + 7) − 7
= 4 × 28 − 7 = 105 > 101.

Hence r = 3. This leads to k1 + k2 + k3 = 26 and one can choose distinct positive


integers k1 , k2 , k3 in several ways.

———-00———-

4
Regional Mathematical Olympiad-2010
Problems and Solutions

1. Let ABCDEF be a convex hexagon in which the diagonals AD, BE, CF are concurrent
at O. Suppose the area of traingle OAF is the geometric mean of those of OAB and
OEF ; and the area of triangle OBC is the geometric mean of those of OAB and OCD.
Prove that the area of triangle OED is the geometric mean of those of OCD and OEF .

D Solution: Let OA = a, OB = b, OC = c,
E OD = d, OE = e, OF = f , [OAB] =
u x, [OCD] = y, [OEF ] = z, [ODE] = u,
d [OF A] = v and [OBC] = w. We are given
e
z y that v 2 = zx, w2 = xy and we have to
f O prove that u2 = yz.
F c C Since ∠AOB = ∠DOE, we have
v a w
b 1
u de sin ∠DOE de
x = 2 = .
x 1 ab
A B ab sin ∠AOB
2
Similarly, we obtain
v fa w bc
= , = .
y cd z ef
Multiplying, these three equalities, we get uvw = xyz. Hence

x2 y 2 z 2 = u2 v 2 w2 = u2 (zx)(xy).

This gives u2 = yz, as desired.

2. Let P1 (x) = ax2 − bx − c, P2 (x) = bx2 − cx − a, P3 (x) = cx2 − ax − b be three quadratic


polynomials where a, b, c are non-zero real numbers. Suppose there exists a real number
α such that P1 (α) = P2 (α) = P3 (α). Prove that a = b = c.

Solution: We have three relations:

aα2 − bα − c = λ,
bα2 − cα − a = λ,
cα2 − aα − b = λ,

where λ is the common value. Eliminating α2 from these, taking these equations pair-
wise,we get three relations:

(ca − b2 )α − (bc − a2 ) = λ(b − a), (ab − c2 )α − (ca − b2 ) = λ(c − b),


(bc − a2 ) − (ab − c2 ) = λ(a − c).

Adding these three, we get

(ab + bc + ca − a2 − b2 − c2 )(α − 1) = 0.

(Alternatively, multiplying above relations respectively by b − c, c − a and a − b, and


adding also leads to this.) Thus either ab + bc + ca − a2 − b2 − c2 = 0 or α = 1. In the
first case
1 
0 = ab + bc + ca − a2 − b2 − c2 = (a − b)2 + (b − c)2 + (c − a)2
2

1
shows that a = b = c. If α = 1, then we obtain

a − b − c = b − c − a = c − a − b,

and once again we obtain a = b = c.

3. Find the number of 4-digit numbers(in base 10) having non-zero digits and which are
divisible by 4 but not by 8.

Solution: We divide the even 4-digit numbers having non-zero digits into 4 classes:
those ending in 2,4,6,8.

(A) Suppose a 4-digit number ends in 2. Then the second right digit must be odd in
order to be divisible by 4. Thus the last 2 digits must be of the form 12, 32,52,72
or 92. If a number ends in 12, 52 or 92, then the previous digit must be even in
order not to be divisible by 8 and we have 4 admissible even digits. Now the left
most digit of such a 4-digit number can be any non-zero digit and there are 9 such
ways, and we get 9 × 4 × 3 = 108 such numbers. If a number ends in 32 or 72,
then the previous digit must be odd in order not to be divisible by 8 and we have
5 admissible odd digits. Here again the left most digit of such a 4-digit number can
be any non-zero digit and there are 9 such ways, and we get 9 × 5 × 2 = 90 such
numbers. Thus the number of 4-digit numbers having non-zero digits, ending in 2,
divisible by 4 but not by 8 is 108 + 90 = 198.
(B) If the number ends in 4, then the previous digit must be even for divisibility by 4.
Thus the last two digits must be of the form 24, 44, 54, 84. If we take numbers
ending with 24 and 64, then the previous digit must be odd for non-divisibility by
8 and the left most digit can be any non-zero digit. Here we get 9 × 5 × 2 = 90 such
numbers. If the last two digits are of the form 44 and 84, then previous digit must
be even for non-divisibility by 8. And the left most digit can take 9 possible values.
We thus get 9 × 4 × 2 = 72 numbers. Thus the admissible numbers ending in 4 is
90 + 72 = 162.
(C) If a number ends with 6, then the last two digits must be of the form 16,36,56,76,96.
For numbers ending with 16, 56,76, the previous digit must be odd. For numbers
ending with 36, 76, the previous digit must be even. Thus we get here (9 × 5 × 3) +
(9 × 4 × 2) = 135 + 72 = 207 numbers.
(D) If a number ends with 8, then the last two digits must be of the form 28,48,68,88. For
numbers ending with 28, 68, the previous digit must be even. For numbers ending
with 48, 88, the previous digit must be odd. Thus we get (9 × 4 × 2) + (9 × 5 × 2) =
72 + 90 = 162 numbers.

Thus the number of 4-digit numbers, having non-zero digits, and divisible by 4 but not
by 8 is
198 + 162 + 207 + 162 = 729.

Alternative Solution:. If we take any four consecutive even numbers and divide them
by 8, we get remainders 0,2,4,6 in some order. Thus there is only one number of the
form 8k + 4 among them which is divisible by 4 but not by 8. Hence if we take four even
consecutive numbers

1000a + 100b + 10c + 2, 1000a + 100b + 10c + 4,


1000a + 100b + 10c + 6, 1000a + 100b + 10c + 8,

there is exactly one among these four which is divisible by 4 but not by 8. Now we
can divide the set of all 4-digit even numbers with non-zero digits into groups of 4 such

2
consecutive even numbers with a, b, c nonzero. And in each group, there is exactly one
number which is divisible by 4 but not by 8. The number of such groups is precisely
equal to 9 × 9 × 9 = 729, since we can vary a, b.c in the set {1, 2, 3, 4, 5, 6, 7, 8, 9}.

4. Find three distinct positive integers with the least possible sum such that the sum of the
reciprocals of any two integers among them is an integral multiple of the reciprocal of
the third integer.

Solution: Let x, y, z be three distinct positive integers satisfying the given conditions.
We may assume that x < y < z. Thus we have three relations:
1 1 a 1 1 b 1 1 c
+ = , + = , + = ,
y z x z x y x y z
for some positive integers a, b, c. Thus
1 1 1 a+1 b+1 c+1
+ + = = = = r,
x y z x y z
say. Since x < y < z, we observe that a < b < c. We also get
1 r 1 r 1 r
= , = , = .
x a+1 y b+1 z c+1
Adding these, we obtain
1 1 1 r r r
r= + + = + + ,
x y z a+1 b+1 c+1
or
1 1 1
+ + = 1. (1)
a+1 b+1 c+1
Using a < b < c, we get
1 1 1 3
1= + + < .
a+1 b+1 c+1 a+1
Thus a < 2. We conclude that a = 1. Putting this in the relation (1), we get
1 1 1 1
+ =1− = .
b+1 c+1 2 2
Hence b < c gives
1 2
< .
2 b+1
Thus b + 1 < 4 or b < 3. Since b > a = 1, we must have b = 2. This gives
1 1 1 1
= − = ,
c+1 2 3 6
or c = 5. Thus x : y : z = a + 1 : b + 1 : c + 1 = 2 : 3 : 6. Thus the required numbers
with the least sum are 2,3,6.

Alternative Solution: We first observe that (1, a, b) is not a solution whenever 1 <
1 1 l
a < b. Otherwise we should have + = = l for some integer l. Hence we obtain
a b 1
a+b
= l showing that a b and b a. Thus a = b contradicting a 6= b. Thus the least
ab
number should be 2. It is easy to verify that (2, 3, 4) and (2, 3, 5) are not solutions and
(2, 3, 6) satisfies all the conditions.(We may observe (2, 4, 5) is also not a solution.) Since
3 + 4 + 5 = 12 > 11 = 2 + 3 + 6, it follows that (2, 3, 6) has the required minimality.

3
5. Let ABC be a triangle in which ∠A = 60◦ . Let BE and CF be the bisectors of the
angles ∠B and ∠C with E on AC and F on AB. Let M be the reflection of A in the
line EF . Prove that M lies on BC.

A Solution: Draw AL ⊥ EF and extend


it to meet AB in M . We show that AL =
LM . First we show that A, F, I, E are con-
cyclic. We have
L E ∠A
F ∠BIC = 90◦ + = 90◦ + 30◦ = 120◦ .
2
I
Hence ∠F IE = ∠BIC = 120◦ . Since
∠A = 60◦ , it follows that A, F, I, E are
concyclic. Hence ∠BEF = ∠IEF =
∠IAF = ∠A/2. This gives
B M C
∠B ∠A
∠AF E = ∠ABE + ∠BEF = + .
2 2
Since ∠ALF = 90◦ , we see that
∠B ∠A ∠C
∠F AM = 90◦ − ∠AF E = 90◦ − − = = ∠F CM.
2 2 2
This implies that F, M, C, A are concyclic. It follows that
∠C
∠F M A = ∠F CA = = ∠F AM.
2
Hence F M A is an isosceles triangle. But F L ⊥ AM . Hence L is the mid-point of AM
or AL = LM .
" #
n
6. For each integer n ≥ 1, define an = √  , where [x] denotes the largest integer not
n
exceeding x, for any real number x. Find the number of all n in the set {1, 2, 3, . . . , 2010}
for which an > an+1 .

Solution: Let us examine the first few natural numbers: 1,2,3,4,5,6,7,8,9. Here we see
that an = 1, 2, 3, 2, 2, 3, 3, 4, 3. We observe that an ≤ an+1 for all n except when n + 1
is a square in which case an > an+1 . We prove that this observation is valid in general.
Consider the range
m2 , m2 + 1, m2 + 2, . . . , m2 + m, m2 + m + 1, . . . , m2 + 2m.
Let n take values in this range so that n = m2 + r, where 0 ≤ r ≤ 2m. Then we see that

[ n] = m and hence " #  
n m2 + r hri
√  = =m+ .
n m m
Thus an takes the values m, m, m, . . . , m, m + 1, m + 1, m + 1, . . . , m + 1, m + 2, in this
| {z } | {z }
m times m times
range. But when n = (m + 1)2 , we see that an = m + 1. This shows that an−1 > an
whenever n = (m + 1)2 . When we take n in the set {1, 2, 3, . . . , 2010}, we see that the
only squares are 12 , 22 , . . . , 442 (since 442 = 1936 and 452 = 2025) and n = (m + 1)2 is
possible for only 43 values of m. Thus an > an+1 for 43 values of n. (These are 22 − 1,
32 − 1, . . ., 442 − 1.)

———-00———-

You might also like